Author Archives: Hung Nguyen

Đề thi vào lớp 10 chuyên toán Phổ thông Năng khiếu: Năm 2016

ĐỀ BÀI

BÀI 1. 
a) Giải hệ $\left\{\begin{array}{l} (x-2y)(x+my) = m^2-2m-3 \\(y-2x)(y+mx) = m^2-2m-3
\end{array} \right.$ khi $m = -3$ và tìm $m$ để hệ co ít nhất một nghiệm $(x_o, y_o)$ thỏa $x_o > 0, y_o > 0$.
b)  Tìm $a \geq 1$ để phương trình $ax^2 + (1-2a)x + 1-a=0$ có hai nghiệm phân biệt $x_1, x_2$ thỏa $x_2^2 – ax_1 = a^2-a-1$.
BÀI 2.  Cho $x, y$ là hai số nguyên dương mà $x^2 + y^2 + 10$ chia hết cho $xy$.

a) Chứng minh rằng $x, y$ là hai số lẻ và nguyên tố cùng nhau.
b)  Chứng minh $k = \dfrac{x^2+y^2+10}{xy}$ chia hết cho 4 và $k \geq 12$.

BÀI 3.  Biết $x \geq y \geq z, x + y + z =0$ và $x^2 + y^2 + z^2 = 6$.

a) Tính $S = (x-y)^2 + (x-y)(y-z) + (y-z)^2$.
b) Tìm giá trị lớn nhất của $P = |(x-y)(y-z)(z-x)|$.

BÀI 4. Tam giác $ABC$ nhọn có $\angle BAC > 45^o$. Dựng các hình vuông $ABMN, ACPQ$ ($M$ và $C$ khác phía đối với $AB$; $B$ và $Q$ khác phía đối với $AC$). $AQ$ cắt đoạn $BM$ tại $E$ và $NA$ cắt đoạn $CP$ tại $F$.

a) Chứng minh $\triangle ABE \sim \triangle ACF$ và tứ giác $EFQN$ nội tiếp.
b) Chứng minh trung điểm $I$ của $EF$ là tâm đường tròn ngoại tiếp tam giác $ABC$.
c) $MN$ cắt $PQ$ tại $D$, các đường tròn ngoại tiếp các tam giác $DMQ$ và $DNQ$ cắt nhau tại $K$ ($K$ khác $D$), các tiếp tuyến tại $B$ và $C$ của đường tròn ngoại tiếp tam giác $ABC$ cắt nhau tại $J$. Chứng minh các điểm $D, A, K, J$ thẳng hàng.

BÀI 5. Với mỗi số nguyên dương $m$ lớn hơn 1, kí hiệu $s(m)$ là ước nguyên dương lớn nhất của $m$ và khác $m$. Cho số tự nhiên $n > 1$, đặt $n_o = n$ và lần lượt tính các số $n_1 =n_o- s(n_o), n_2 = n_1 – s(n_1), …, n_{i+1} = n_i – s(n_i)$,…. Chứng minh tồn tại số nguyên dương $k$ để $n_k = 1$ và tính $k$ khi $n = 2^{16}.14^{17}$.

Hết

Lời giải. 

Bài 1: 

a) Đây là hệ đối xứng loại 2, nên phương pháp giải là lấy (1) – (2) để có thừa số $x-y$, từ đó giải tiếp.

Chú ý xét trường hợp và điều kiện $x_o > 0, y_o > 0$ để biện luận. Những dạng toán này chú ý tính toán cẩn thận và xét đầy đủ các trường hợp.

b) Là bài dạng  biểu thức nghiệm không đối xứng, có nhiều cách, có thể tính nghiệm theo $m$ từ đó suy ra $m$.

Lời giải.

a) Khi $m = -3$ ta có hệ:

$\left\{\begin{array}{l} (x-2y)(x-3y)=12 \\(y-2x)(y-3x) = 12 \end{array} \right.$

$\Leftrightarrow \left\{\begin{array}{l} x^2-5xy+6y^2=12 (1)\\y^2-5xy+6x^2 = 12(2) \end{array} \right.$

Lấy (1) – (2) ta có $5(y^2-x^2) = 0 \Leftrightarrow x = y, x = -y$.
Với $x= y$ thế vào (1) ta có $x^2 =6 \Leftrightarrow x = \sqrt{6}, y = \sqrt{6}$ hoặc $x=-\sqrt{6}, y = -\sqrt{6}$.
Với $x = -y$ thế vào (1) ta có $x^2 = 1 \Leftrightarrow x = 1, x = -1$. Với $x = 1, y = -1$, với $x=-1, y = 1$.
Vậy hệ phương trình có 4 nghiệm.
Hệ có thể viết lại $\left\{\begin{array}{l} x^2+(m-2)xy-2my^2 = m^2-2m-3 (1)\\y^2+(m-2)xy-2mx^2= m^2-2m-3(2) \end{array} \right.$

Lấy (1) – (2) ta có $(2m+1)(y^2-x^2) = 0$.
Xét $m = \dfrac{-1}{2}$ ta có hệ trở thành: $x^2 – \dfrac{5}{2}xy + y^2 + \dfrac{7}{4}=0$, có nghiệm $ (\dfrac{5+\sqrt{2}}{2},2)$ thỏa đề bài.
Xét $m \neq \dfrac{-1}{2}$ ta có $x = y$ hoặc $x = -y$.

Trường hợp $x = -y$ không thỏa đề bài.
Trường hợp $x = y$, thế vào (1) ta có:

$-(m+1)x^2 = m^2-2m-3 = (m+1)(m-3)$.
Nếu $m = -1$ ta có $(x-2y)(x-y) = 0, (y-2x)(y-x) = 0$ có nghiệm thỏa đề bài, chỉ cần chọn $x=1, y=1$.
Nếu $m \neq -1$ ta có $x^2 = 3-m$ để có nghiệm $x_o = y_o > 0$ thì $m < 3$.

Khi đó phương trình có nghiệm $x_0 = \sqrt{3-m}, y_o = \sqrt{3-m}$ thỏa đề bài.

Kết luận $m = \dfrac{-1}{2}, m = -1$ và $m < 3$.

b) Điều kiện để phương trình có hai nghiệm phân biệt $\Delta = (1-2a)^2-4a(1-a) = 8a^2-8a+1 > 0$.
Theo định lý Viete ta có $x_1 + x_2 = \dfrac{2a-1}{a}$, suy ra $ax_1 + ax_2 = 2a – 1$. Suy ra $ax_1 = 2a-1-ax_2$.
Kết hợp giả thiết ta có $x_2^2+ax_2-2a+1=a^2-a-1
\Leftrightarrow x_2^2+ax_2-a^2-a+2=0
\Leftrightarrow ax_2^2+a^2x_2-a^3-a^2+2a=0$ (1).
Mà $x_2$ là nghiệm của phương trình nên ta có $ax_2^2+(1-2a)x_2+1-a = 0 (2)$.
Lấy (1) – (2) ta có $(a^2+2a-1)x_2 = a^3+a^2-3a+1$, mà $a \geq 1$ nên $a^2 + 2a – 1 \neq 0$, suy ra $x_2 = a-1$.
Thế vào phương trình (1) ta có $(a-1)^2+a(a-1)-a^2-a+2 = 0 \Leftrightarrow a=1, a=3$.
Thử lại ta nhận hai giá trị $a = 1, a=3$.

Bài 2.

a) Giả sử trong hai số $x, y$ có một số chẵn, vì vai trò $x, y$ như nhau nên có thể giả sử $x$ chẵn. Suy ra $x^2 + y^2 + 10$ chia hết cho 2, suy ra $y$ chẵn. Khi đó $x^2 + y^2 + 10$ chia hết cho 4, suy ra 10 chia hết cho 4 vô lý.
Vậy trong hai số đều là số lẻ.
Đặt $d= (x,y)$, $x= d.x’, y = d.y’$ ta có $x^2 + y^2 + 10 = d^2(x’^2 + y’^2) + 10$ chia hết cho $d^2x’y’$. Suy ra 10 chia hết cho $d^2$. Suy ra $d= 1$. Vậy $x, y$ nguyên tố cùng nhau.

b) Đặt $x = 2m + 1, y = 2n + 1$, suy ra $k = \dfrac{4(m^2+m+n^2+n+3}{(2m+1)(2n+1)}$, ta có $4, (2m+1).(2n+1)$ nguyên tố cùng nhau. Suy ra $m^2 + n^2 +m+n+3$ chia hết cho $(2m+1)(2n+1)$. Từ đó ta có $k$ chia hết cho 4. Chứng minh $k \geq 12$ bằng hai cách.
Cách 1: Ta có $x^2 + y^2 + 10 = kxy$.
Nếu trong hai số $x, y$ có một số chia hết cho 3, giả sử $x$ chia hết cho 3. Ta có $y^2 + 10$ chia hết cho 3 vô lý vì $y^2 $ chia 3 dư 0 hoặc dư 1.
Vậy $x, y$ không chia hết cho 3, suy ra $x^2 + y^2 + 10$ chia hết cho 3 và $3, xy$ nguyên tố cùng nhau. Do đó $k$ chia hết cho 3.
Do đó $k$ chia hết cho 12, vậy $k\geq 12$.
Cách 2: Xét $k=4$ ta có $x^2 + y^2 + 10 = 4xy$ () $\Leftrightarrow (x-2y)^2 = 3y^2 – 10$.
Ta có $(x-2y)^2$ chia 3 dư 0 hoặc 1 mà $3y^2-10$ chia 3 dư 2, nên phương trình (
) không có nghiệm nguyên dương.
Xét $k=8$ ta có $x^2 + y^2 + 10 = 8xy (*)\Leftrightarrow (x-4y)^2 = 15y^2 -10$.
Ta có $(x-4y)^2$ chia 3 dư 0 hoặc 1 mà $15y^2-10$ chia 3 dư 2 nên (**) không có nghiệm nguyên dương.
Vậy $k \geq 12$.

Bài 3. Bài này là bài bdt khó, nhưng câu a đã gợi ý để làm câu b, chú ý các bdt phụ quan trọng.

a) Ta có $(x+y+z)^2 = x^2+y^2+z^2 + 2(xy+yz+xz)$. Suy ra $xy + yz + xz = -3$.
Ta có $S = (x-y)^2 + (x-y)(y-z) + (y-z)^2 $

$= x^2 -2xy+y^2+xy-y^2+yz-xz+y^2-2yz + z^2$

$= x^2+y^2+z^2-yx-yz-xz = 9$.

b) Ta có thể chứng minh trực tiếp không qua câu a) như sau:

$(x-y)(y-z) \leq \dfrac{1}{3}((x-y)^2+(x-y)(y-z) + (y-z)^2) = 3$. Suy ra $P \leq 3|x-z|$.
Ta có $|x-z| \leq \sqrt{2(x^2+z^2)}\leq \sqrt{2(x^2+y^2+z^2)}= \sqrt{12}$. Suy ra $P \leq 3\sqrt{12} = 6\sqrt{3}$.
Đẳng thức xảy ra khi $x = \sqrt{3}, y =0, z = -\sqrt{3}$.

Vậy giá trị lớn nhất của P là $6\sqrt{3}$ khi $x = \sqrt{3}, y =0, z = -\sqrt{3}$

Ngoài ra ta có thể áp dụng câu a: Đặt $a = x-y, b = y-z$ ta có $a^2+b^2+ab = 9$, cần tìm giá trị lớn nhất của $P = ab(a+b)$.

Áp dụng $ab \leq \dfrac{1}{4} (a+b)^2$ và $a^2+b^2+ab \geq \dfrac{3}{4} (a+b)^2$. Ta có điều cần chứng minh.

Bài 4. Đây là bài hình khó và dài, các em chú ý hình vẽ cụ thể là góc, vẽ hình chính xác. 

Tránh dùng các kiến thức cấp 3: phương tích trục đẳng phương,…

a) Ta có $\angle EAB + \angle BAC = 90^\circ, \angle FAC + \angle BAC = 90^\circ$. Suy ra $\angle EAB = \angle FAC$.
Mặt khác có $\angle ABE = \angle ACF = 90^\circ$. Suy ra $\triangle ABE \backsim \triangle ACF$.
Suy ra $AE\cdot AC = AF\cdot AB$ mà $ AC = AQ, AB = AN$. Suy ra $AE\cdot AQ = AN\cdot AF$. Suy ra tứ giác $QNEF$ nội tiếp.
b) Cách 1: Gọi $T$ là giao điểm của $MB$ và $CP$. Ta có $ABTC$ nội tiếp và $AT$ là đường kính của đường tròn ngoại tiếp tam giác $ABC$. Mặt khác ta có $AF|| ET, AE|| FT$ nên $AETF$ là hình bình hành. Suy ra trung điểm $EF$ cũng là trung điểm $AT$. Do đó trung điểm $I$ của $EF$ là tâm đường tròn ngoại tiếp tam giác $ABC$.
Cách 2: Xét hình thang $AEBF$, gọi $X$ là trung điểm của $AB$ khi đó $IX$ thuộc đường trung bình của hình thang, suy ra $IX || BE$ hay $IX$ vuông góc $AB$ vậy $IX$ là trung trực của đoạn $AB$. Chứng minh tương tự thì $I$ cũng thuộc trung trực đoạn $AC$. Vậy $I$ là tâm ngoại tiếp của tam giác $ABC$.

b) $DA$ cắt $EF$ tại $K’$ ta có $\angle NFK’ = \angle NQA$ (vì $NQFE$ nội tiếp). Mà $\angle NQA = \angle NDA$(vì $AQDN$ nội tiếp). Suy ra $\angle NDA = \angle AFK’$.
Suy ra $NDFK’$ nội tiếp. Chứng minh tương tự ta có $DQK’E$ nội tiếp.
Do đó $K’$ là giao điểm của đường tròn ngoại tiếp hai tam giác $DQM$ và $DPN$. Vậy $K’ \equiv K$. Suy ra $D, A, K$ thẳng hàng.
Ta có $\angle BKE = \angle EAB = \angle CAF = \angle CKF$. Suy ra $\angle BKC = 180^\circ – 2 \angle BKE = 2(90^\circ – \angle EAB) = 2\angle BAC = \angle BIC$. Suy ra $BKIC$ nội tiếp. Mà $IBJC$ nội tiếp, suy ra và $JB = JC$ nên $\angle BKJ = \angle CKJ$. Hay $KJ$ là phân giác $\angle BKC$.
Mặt khác $\angle BKA = 180^\circ – \angle AEB = 180^\circ – \angle AFC = \angle AKC$. Suy ra tia đối của tia $KA$ cũng là phân giác của $\angle BKC$. Do đó $A, K, J$ thẳng hàng.
Vậy 4 điểm $D, A, K, J$ thẳng hàng.

Bài 5. Đây là bài toán lạ và khá hay, sử dụng đơn biến.

Ta có $s(n_i) < n_i$, suy ra $n_i – s(n_i) \geq 1$. Suy ra $n_{i+1} \geq 1$. Do đó $n_i \geq 1$ với mọi $i = 1, 2, …$.
Mặt khác $n_{i+1} = n_i – s(n_i) < n_i$ với mọi $i$. Suy ra $n=n_o > n_1 > n_2 > …>…$.
Nếu không tồn tại $n_k$ để $n_k = 1$ ta xây dựng được dãy vô hạn các số nguyên dương giảm và nhỏ hơn $n$ (vô lý) vì số các số nhỏ hơn $n$ là bằng $n-1$.
Vậy tồn tại $k$ sao cho $n_k = 1$.
Với $n=2^{16}.14^{17} = 2^{33}.7^{17}$, ta có $n_1 = 2^{33}7^{17} – 2^{32}.7^{17}= 2^{32}.7^{17}$.\
$n_2 = 2^{31}.7^{17}$.
Tiếp tục ta có $n_{33} = 7^{17}$.
Đặt $m_o= 7^{17}$ ta có $m_1 = 6.7^{16}$, $m_2 = 3.7^{16}, m_3 = 2.7^{16}, m_4 = 7^{16}$. Tương tự ta có $m_8 = 7^{15}$,…,$m_{68} = 7^0 = 1$.
Vậy $k = 33 + 68 = 101$.

Đáp án và bình luận thi vào lớp 10 PTNK năm 2013: Đề chuyên toán

ĐỀ BÀI

BÀI 1. Cho phương trình $x^2-4mx+m^2-2m+1=0$ (1) với m là tham số .

a) Tìm m sao cho phương trình (1) có hai nghiệm phân biệt. Chứng minh rằng khi đó hai
nghiệm không thể trái dấu.
b)  Tìm $m$ để phương trình có hai nghiệm $x_1, x_2$ thỏa $|x_1 -x_2| =1$.

BÀI 2.  Giải hệ phương trình $\left\{ \begin{array}{l}
3{x^2} + 2y + 1 = 2z\left( {x + 2} \right)\\
3{y^2} + 2z + 1 = 2x\left( {y + 2} \right)\\
3{z^2} + 2x + 1 = 2y\left( {z + 2} \right)
\end{array} \right.$

BÀI 3. Cho $x, y$ là hai số không âm thỏa $x^3+y^3 < x- y$.

a) Chứng minh rằng $y \leq x \leq 1$.
b) Chứng minh rằng $x^3+y^3 \leq x^2 + y^2 \leq 1$.

BÀI 4.  Cho $M = a^2 + 3a + 1$ với $a$ là số nguyên dương.

a) Chứng minh rằng mọi ước của $M$ đều là số lẻ.
b) Tìm $a$ sao cho $M$ chia hết cho 5. Với những giá trị nào của $a$ thì $M$ là lũy thừa của 5?

BÀI 5.  Cho tam giác $ABC$ có góc $\angle A = 60^o$ , đường tròn $(I)$ nội tiếp tam giác (với tâm I) tiếp xúc với các cạnh $BC,CA,AB$ lần lượt tại $D,E,F$. Đường thẳng $ID$ cắt $EF$ tại $K$, đường thẳng qua $K$ và song song với $BC$ cắt $AB,AC$ theo thứ tự tại $M,N$.

a) Chứng minh rằng các tứ giác $IFMK$ và $IMAN$ nội tiếp .
b) Gọi $J$ là trung điểm cạnh $BC$.Chứng minh rằng ba điểm $A,K,J$ thẳng hàng.
c) Gọi $r$ là bán kính của dường tròn $(I)$ và $S$ là diện tích tứ giác $IEAF$.Tính $S$ theo $r$ và
chứng minh $S_{IMN} \geq \dfrac{S}{4}$ ($S_{IMN}$ là diện tích tam giác $IMN$).

BÀI 6.  Trong một kỳ thi, 60 thí sinh phải giải 3 bài toán. Khi kết thúc kỳ thi , người ta nhận
thấy rằng: Với hai thí sinh bất kỳ luôn có ít nhất một bài toán mà cả hai thí sinh đó đều giải
được. Chứng minh rằng :

a) Nếu có một bài toán mà mọi thí sinh đều không giải được thì phải có một bài toán khác mà
mọi thí sinh đều giải được .
b) Có một bài toán mà có ít nhất 40 thí sinh giải được.

LỜI GIẢI

Nhìn vào đề này thấy độ phức tạp nhẹ nhàng, các câu dễ có thể một phát ăn ngay là bài 1, 3a, 4a, 4b ý đầu, 5a.

Tiếp theo là các câu khó hơn như 2,3b ý sau, 5b, 5c và khó nhằn nhất có lẽ là bài tổ hợp.

Bài hình trong đề này là một bài rất quen thuộc, do đó việc giải lại các bài toán đã học là một việc quan trọng. Chú ý những lỗi suy luận trong làm bài, các em tự làm và tự đánh giá điểm để xem được nhiêu điểm nhé, đáp án sẽ có sau vài ngày nữa.

Bài 1. (1,5 điểm) 

a) Phương trình có hai nghiệm phân biệt khi và chỉ khi $\Delta ‘ = 3m^2+2m-1> 0 \Leftrightarrow m > \dfrac{1}{3}$ hoặc $m < – 1$. Khi đó tích hai nghiệm của phương trình $x_1x_2 = (m-1)^2 \geq 0$ nên phương trình không thể có hai nghiệm trái dấu.

b) Điều kiện để phương trình có hai nghiệm $x_1, x_2$ không âm:

$\Delta’ = 3m^2+2m-1\geq 0; S = x_1+x_2 \geq 0; P=x_1x_2 = m^2-2m+1 \geq 0$

$\Leftrightarrow m \geq \dfrac{1}{3} $
Ta có $|\sqrt{x_1}-\sqrt{2}| = 1 $
$\Leftrightarrow x_1 + x_2 – \sqrt{x_1x_2} = 1 $
$\Leftrightarrow 4m – 2\sqrt{m^2-2m+1} = 1 $
$\Leftrightarrow m = \dfrac{1}{2} (n), m = \dfrac{-1}{2} (l)$.

Bình luận Nhiều bạn xét $P \geq 0$ suy ra phương trình có hai nghiệm cùng dấu, cái này là suy luận sai, vì còn trường hợp bằng 0, tốt nhất là dùng phản chứng.

Bài 2. (1 điểm) Cộng ba phương trình lại ta có:
$3(x^2+y^2+z^2) + 2(x+y+z)+3 = 2(xy+yz+zx) + 4(x+y+z)$

$\Leftrightarrow 3(x^2+y^2+z^2)-2(xy+yz+xz) – 2(x+y+z)+3 = 0$

$\Leftrightarrow (x-y)^2+(y-z)^2+(z-x)^2 + (x-1)^2+(y-1)^2+(z-1)^2 = 0$
$\Leftrightarrow \left\{\begin{array}{l}x=1\\y=1\\z=1
\end{array} \right.$
Thử lại thấy $(1, 1,1)$ là nghiệm của hệ.

Bình luận: Bài này hệ hoái vị vòng quanh, bất đẳng thức là một trong những cách hay dùng.

Bài 3. (1,5 điểm) 

a) Ta có $x – y \geq x^3 + y^3 \geq 0$, suy ra $x \geq y$.
Ta có $x \geq y + y^3 + x^3 \geq x^3$, suy ra $x(1-x)(1+x) \geq 0$. \Suy ra $0\leq x \leq 1$.
Do đó $0 \leq y \leq x \leq 1$.
b) Từ câu a ta có $0 \leq y \leq x \leq 1$, suy ra $x^3 \leq x^2, y^3 \leq y^2$. Suy ra $x^3+y^3 \leq x^2+y^2$.
Ta có $x – y \geq x^3+y^3 \geq x^3-y^3 \geq 0$.
Suy ra $x^2+y^2+xy \leq 1$, suy ra $x^2+y^2 \leq 1$.
Vậy $x^3+y^3\leq x^2+y^2 \leq 1$.

Bình luận: Đây là bất đẳng thức tương đối dễ, chỉ dùng các biến đổi đơn giản, tuy vậy để làm được ý cuối trong điều kiện phòng thi thì không đơn giản.

Bài 4. (1,5 điểm) 

a)Ta có $M = a^2 + 3a + 1 = a(a+1) + 2a + 1$. Mà $a(a+1)$ là tích hai số tự nhiên liến tiếp nên chia hết cho 2, suy ra $M = a(a+1) + 2a +1$ là số lẻ, do đó mọi ước của $M$ đều là số lẻ.
b) Giả sử $M = a^2 + 3a + 1$ chia hết cho 5. Mà $M = (a-1)^2 + 5a$ nên $(a-1)^2$ chia hết cho 5. Suy ra $a = 5k + 1$ ($k$ là số tự nhiên).
Thử lại thấy với $a = 5k + 1$ thì M chia hết cho 5.
Giả sử $M = (a-1)^2+ 5a = 5^n$.
Nếu $n \geq $ ta có $M$ chia hết cho 25.
Từ M chia hết cho 5, tương tự trên ta có $a = 5k + 1$.
Khi đó $M = 25k^2 + 25k + 5 = 5(5k^2+5k+1)$. Ta có $5k^2 + 5k + 1$ không chia hết cho 5 nên M không chia hết cho 25. (mâu thuẫn).
Nếu $n = 1$. Khi đó $k = 0, a= 1$ và $A=5$ thỏa đề bài.
Đáp số $a = 1$.

Bình luận: Bài này thực chất là bài phương trình nghiệm nguyên, cách hay sử dụng là đồng dư, và đưa ra điều kiện của $a$, ta cũng có thể thử vài giá trị để đoán được nghiệm, từ đó cho ra cách giải.

Bên cạnh đó, nắm chắc một chút các phương pháp chia hết như biến đổi thành tổng.

Bài 5.  (3 điểm) 

a) Do $MN|| BC$ nên $IK \bot MN$. Do $\angle IKN = \angle IFM = 90^\circ$ nên tứ giác $IFMK$ nội tiếp. Tam giác $AEF$ đều nên $\angle KFI = 30^\circ$. Từ đó $\angle IMN = \angle KFI = \angle IAN = 30^\circ$ nên tứ giác $IMAN$ nội tiếp.
b) Ta có $\angle IMN = \angle INM = 30^\circ$ nên tam giác $IMN$ cân tại $I$.
Lại có $IK \bot MN$ nên $K$ là trung điểm của $MN$.
Gọi $J’$ là giao điểm của $AK$ và $BC$, ta có $\dfrac{MK}{BJ’} = \dfrac{AK}{AJ’} = \dfrac{NK}{CJ’}$ mà $MK = NK$ nên $BJ’ = CJ’$. Suy ra $J’$ là trung điểm của $BC$. Suy ra $J \equiv J’$, do đó $A, K, J$ thẳng hàng.
b) Ta có $AE = AF = r\sqrt{3}$, suy ra $S = 2S_{IAF} = 2.\dfrac{1}{2}IF\cdot AF = r^2 \sqrt{3}$.

Ta chứng minh được $S_{IEF} = \dfrac{1}{4}S$.
Các tam giác $IMN$ và $IEF$ cân tại $I$ có $\angle IMN = \angle IEF$ nên đồng dạng. Do đó $\dfrac{S_{IMN}}{S_{IEF}} = \dfrac{IM^2}{IF^2} \geq 1$ (do $IM \geq IF$). Suy ra $S_{IMN} \geq S_{IEF} = \dfrac{S}{4}$.
Dấu bằng xảy ra khi $M \equiv F$ hay tam giác $ABC$ là tam giác đều.

Bình luận. Đây là một mô hình quen thuộc của đường tròn nội tiếp, hầu hết các bạn đã gặp bài toán này, do đó nắm chắc các bài toán là một lợi thế.

Bài 6. (1,5 điểm) 

a) Kí hiệu các bài toán là BT1, BT2, BT3.
Từ giả thiết suy ra rằng mọi thí sinh đều giải được ít nhất một bài toán.
Ta giả sử, mọi thí sinh đều không giải được BT1. Khi đó mọi thí sinh đều giải được BT2 hoặc BT3. Nếu có một thí sinh chỉ giải được 1 bài toán, giả sử đó là bài toán 2. Khi đó theo đề bài thì mọi thí sinh khác đều giải được bài toán 2. Vậy mọi thí sinh đều giải được bài toán 2. Còn nếu tất cả các thí sinh đều giải được 2 bài toán thì cũng thỏa.

b) Ta xét hai trường hợp:
TH1: Có một thí sinh nào đó giải đúng một bài toán, theo câu a thì mọi thí sinh đều giải được bài toán đó, ta có điều cần chứng minh.
TH2: Mọi thí sinh đều giải được ít nhất 2 bài toán. Gọi $a$ là số thi sinh giải được cả 3 bài toán, $b$ là số thí sinh giải được BT1 và BT2, $c$ là số thí sinh giải được BT2 và BT3, $d$ là số thí sinh giải được BT1 và BT3.
Ta có $a + b+ c+ d = 60$.
Nếu $b, c, d > 20$, suy ra $b+c+d > 60$ vô lý. Do đó có một trong ba số $b, c, d$ phải nhỏ hơn hoặc bằng 20. Giả sử là $b \leq 20$. Suy ra $a+c+d \geq 40$.

Hay số thí sinh giải được bài BT3 không ít hơn 40. Điều cần chứng minh.

Bình luận: Đây là bài tổ hợp vừa phải, câu a, chỉ cần đọc kĩ giả thiết là làm được.

Câu b, là biểu đồ venn có thể suy nghĩ đến khi cần phân ra các tập rời nhau.

Bên cạnh đó phản chứng là phương pháp được sử dụng.

Nhìn chung đề này có nhiều câu dễ và quen thuộc, với những câu đó phải làm trước và làm thật chắc, khi đó mới có nhiều thời gian làm các câu khó.

Bổ đề về số mũ đúng

BỔ ĐỀ VỀ SỐ MŨ ĐÚNG

(Thầy Nguyễn Ngọc Duy giáo viên trường PTNK TP Hồ Chí Minh)

Bổ đề số mũ đúng của một số nguyên là một hướng tiếp cận khá mới đối với các bài toán sơ cấp. Nó cung cấp một công cụ khá hữu hiệu để giải các phương trình Diophante hoặc các bài toán chia hết liên quan đến số mũ. Trong bài viết này tôi sẽ cố gắng mang đến một cái nhìn thật sơ cấp và tự nhiên đến vấn đề, trang bị thêm kiến thức và kĩ năng cho các các em học sinh để giải quyết các bài toán số học. Đặc biệt, ta sẽ dùng bổ đề số mũ đúng để giải quyết một số trường hợp đặc biệt của định lí lớn Fermat.

1. Kiến thức cần nhớ

Định nghĩa 1.1: Cho $\left( a,n \right)=1$. Kí hiệu cấp của a theo modulo n là $or{{d}_{n}}\left( a \right)$, là số nguyên dương d nhỏ nhất thỏa $a^d\equiv 1\, \left( \bmod n \right)$.

Tính chất 1.1: Nếu $x$ là số nguyên dương thỏa mãn $a^x \equiv 1\, \left( \bmod n \right)$ thì $or{{d}_{n}}\left( a \right)|x$.

Định nghĩa 1.2: Cho $p$ là số nguyên tố, $x$ là số nguyên bất kì, kí hiệu $v_p \left( x \right)=n$ nếu $x$ chia hết cho $p^n$ nhưng không chia hết cho $p^{n+1}$ .

Tính chất 1.2: Với $a,b$ là các số nguyên và $n$ là số nguyên dương thì:

  • $v_p \left( ab \right)=v_p \left( a \right)+v_p \left( b \right)$.
  • Nếu $p|a$ thì $v_p(a) >0.$
  • $v_p \left( a^n \right)=n v_p \left( a \right)$.
  • $v_p \left( a+b \right) \ge \min \left\{ v_p \left( a \right), v_p \left( b \right) \right\}$. Đẳng thức xảy ra chẳng hạn khi $v_p(a) \neq v_p(b).$
  • $v_p(\text{gcd}(a,b)) = \min(v_p(a), v_p(b))$ và $v_p(\text{lcm}(a,b)) = \max(v_p(a), v_p(b)).$

Định lý 1.1: Bổ đề số mũ đúng. Cho $p$ là số nguyên tố lẻ; $a,b$ không chia hết cho $p$

$i)$  Nếu $a-b$ chia hết cho p thì $v_p \left( a^n – b^n \right)=v_p \left( a-b \right)+v_p \left( n \right)$.

$ii)$  Nếu $a+b$ chia hết cho $p, n$ lẻ thì $v_p\left( a^n+b^n \right)=v_p\left( a+b \right)+v_p \left( n \right)$.

$iii)$  Nếu $a, b$ lẻ thì $v_2 \left( a^n – b^n \right)=v_2 \left( \dfrac{x^2 – y^2}{2} \right) + v_2 \left( n \right)$.

Chứng minh
  • Trước tiên, ta chứng minh: ${{v}_{p}}\left( {{a}^{p}}-{{b}^{p}} \right)={{v}_{p}}\left( a-b \right)+1$ $(*)$. Ta có:

$${{a}^{p}}-{{b}^{p}}=\left( a-b \right)\left( {{a}^{p-1}}+{{a}^{p-2}}b+…+a{{b}^{p-2}}+{{b}^{p-1}} \right).$$

Do $a\equiv b\left( \bmod p \right)$ nên ${{a}^{p-1}}+{{a}^{p-2}}b+…+a{{b}^{p-2}}+{{b}^{p-1}}\equiv p.{{a}^{p-1}}\equiv 0\left( \bmod p \right)$.

Suy ra : ${{a}^{p-1}}+{{a}^{p-2}}b+…+a{{b}^{p-2}}+{{b}^{p-1}}$ chia hết cho $p$  $(1)$.

Ta chứng minh tiếp $${{a}^{p-1}}+{{a}^{p-2}}b+…+a{{b}^{p-2}}+{{b}^{p-1}} \text{không chia hết cho } {{p}^{2}}. $$

Thật vậy, do $a\equiv b\left( \bmod p \right)$ nên $a=b+kp$ . Sử dụng khai triển nhị thức Newton ta có

$ {{a}^{p-1}}+{{a}^{p-2}}b+\cdots+{{b}^{p-1}}$

$\equiv \left[ \left( p-1 \right)kp{{b}^{p-2}}+{{b}^{p-1}} \right]+\left[ \left( p-2 \right)kp{{b}^{p-2}}+{{b}^{p-1}} \right]+  \cdots+\left[ kp{{b}^{p-2}}+{{b}^{p-1}} \right]+{{b}^{p-1}}\left( \bmod {{p}^{2}} \right) $

$\equiv \dfrac{p\left( p-1 \right)}{2}kp{{b}^{n-2}}+p.{{b}^{p-1}}$

$\equiv p{{b}^{p-1}}\left( \bmod {{p}^{2}} \right) $

Theo giả thiết thì $b$ không chia hết cho $p$ nên $p{{b}^{p-1}}$ không chia hết cho ${{p}^{2}}$. Do đó ${{a}^{p-1}}+{{a}^{p-2}}b+\cdots+a{{b}^{p-2}}+{{b}^{p-1}}$ cũng không chia hết cho ${{p}^{2}}$  $(2)$.

Từ $(1), (2)$ ta có: ${{v}_{p}}\left( {{a}^{p-1}}+{{a}^{p-2}}b+\cdots+a{{b}^{p-2}}+{{b}^{p-1}} \right)=1$.

Vậy ${{v}_{p}}\left( {{a}^{p}}-{{b}^{p}} \right)={{v}_{p}}\left( a-b \right)+1$.

  • Tương tự, ta cũng có: nếu m không chia hết cho p thì ${{v}_{p}}\left( {{a}^{m}}-{{b}^{m}} \right)={{v}_{p}}\left( a-b \right)$ $(**)$.

Ta quay lại định lí. Đặt ${{v}_{p}}\left( n \right)=k\Rightarrow n={{p}^{k}}.m$, với $\left( m,p \right)=1$.

Áp dụng $(*)$ và $(**)$ ta có:

${{v}_{p}}\left( {{a}^{n}}-{{b}^{n}} \right)  ={{v}_{p}}\left( {{\left( {{a}^{{{p}^{k-1}}.m}} \right)}^{p}}-{{\left( {{b}^{{{p}^{k-1}}.m}} \right)}^{p}} \right) $

$={{v}_{p}}\left( {{a}^{{{p}^{k-1}}.m}}-{{b}^{{{p}^{k-1}}.m}} \right)+1=\ldots={{v}_{p}}\left( {{a}^{m}}-{{b}^{m}} \right)+k $

$={{v}_{p}}\left( a-b \right)+{{v}_{p}}\left( n \right).$

Vậy ta đã chứng minh xong phần $i)$ của định lí.

Vì $n$ lẻ nên thay $b$ bởi $-b$ trong i. ta được ${{v}_{p}}\left( {{a}^{n}}+{{b}^{n}} \right)={{v}_{p}}\left( {{a}^{n}}-{{\left( -b \right)}^{n}} \right)={{v}_{p}}\left( a+b \right)+{{v}_{p}}\left( n \right)$

Vậy ta đã chứng minh xong phần $ii)$ của định lí. Tương tự cách làm trong $i)$ ta cũng có kết quả $iii)$.

Như vậy ta đã chứng minh xong bổ đề số mũ đúng. Sau đây ta sẽ sử dụng bổ đề để giải quyết một bài toán thú vị.

2. Các bài toán áp dụng

Bài toán Fermat lớn: Cho $n$ là số tự nhiên lớn hơn $2.$ Chứng minh rằng phương trình ${{a}^{n}}+{{b}^{n}}={{c}^{n}}$ không có nghiệm nguyên dương.

Bài Toán Fermat lớn là bài toán cực kì thú vị. Nó tồn tại gần bốn thế kỉ, kích thích biết bao nhà toán học thế giới. Bài toán cuối cùng được chứng minh bởi nhà toán học Andrew Wiles vào năm 1993. Và người ta nói rằng sẽ không có phương pháp sơ cấp nào có thể chứng minh bài toán trên. Bài báo sẽ đề cập một trường hợp đặc biệt của bài toán: số $c$ là số nguyên tố. Và chúng ta sẽ giải quyết thông qua bổ đề số mũ đúng.

Bài toán 1: Cho số nguyên lẻ $n>2$, $p$ là số nguyên tố. Chứng minh rằng phương trình $a^n + b^n = p^n$ không có nghiệm nguyên dương.

Giải

Không mất tính tổng quát, giả sử phương trình có nghiệm $a\ge b$ .

$1.$ Nếu $a=1\Rightarrow b=1$, thế vào phương trình suy ra vô lí.

$2.$ Nếu $a=2\Rightarrow b=1;2$.

  • Trường hợp $\left( a,b \right)=\left( 2,2 \right)\Rightarrow p=2$ (vô lí).
  • Trường hợp $\left( a,b \right)=\left( 2,1 \right)\Rightarrow p=3$ , thế vào phương trình ta được ${{3}^{n}}-{{2}^{n}}=1$ , cũng suy ra vô lí.

Vậy bắt buộc $a\ge 3$, mà ${{p}^{n}}>{{a}^{n}}\Rightarrow p>3$ , nên p là số nguyên tố lẻ. Do n lẻ, ta có : $${{p}^{n}}={{a}^{n}}+{{b}^{n}}=\left( a+b \right)\left( {{a}^{n-1}}-{{a}^{n-2}}b+\cdots-a{{b}^{n-2}}+{{b}^{n-1}} \right) $$

Suy ra $p|a+b$ (do $a+b>1$ ). Áp dụng bổ đề số mũ đúng cho $p$, ta có

$${{v}_{p}}\left( {{a}^{n-1}}-{{a}^{n-2}}b+\cdots-a{{b}^{n-2}}+{{b}^{n-1}} \right)={{v}_{p}}\left( {{a}^{n}}+{{b}^{n}} \right)-{{v}_{p}}\left( a+b \right)={{v}_{p}}\left( n \right) $$

Mà ${{a}^{n-1}}-{{a}^{n-2}}b+\cdots-a{{b}^{n-2}}+{{b}^{n-1}}$ là lũy thừa của $p$ nên ta có $$\left( {{a}^{n-1}}-{{a}^{n-2}}b+\cdots-a{{b}^{n-2}}+{{b}^{n-1}} \right)|n.$$

Do ${{a}^{n-1}}-{{a}^{n-2}}b+\cdots-a{{b}^{n-2}}+{{b}^{n-1}}=\frac{1}{2}\left[ {{a}^{n-1}}+{{a}^{n-3}}{{\left( a-b \right)}^{2}}+\cdots+{{b}^{n-3}}{{\left( a-b \right)}^{2}}+{{b}^{n-1}} \right]\ge \dfrac{1}{2}\left( {{a}^{n-1}}+{{b}^{n-1}} \right)$

Vì $a\ge 3$, $n\ge 3$ nên $\frac{1}{2}\left( {{a}^{n-1}}+{{b}^{n-1}} \right)>n$ nên không thể $$\left( {{a}^{n-1}}-{{a}^{n-2}}b+\cdots-a{{b}^{n-2}}+{{b}^{n-1}} \right)|n.$$

Vậy phương trình vô nghiệm khi $p$ là số nguyên tố.

Bài tập 2: Cho số nguyên $n>2$ có ước lẻ khác 1, $p$ là số nguyên tố. Chứng minh rằng phương trình ${{a}^{n}}+{{b}^{n}}={{p}^{n}}$ không có nghiệm nguyên dương.

Giải

Gọi $k>1$ là ước lẻ của $n$, giả sử $n=km$ . Đặt $x={{a}^{m}};y={{b}^{m}}$. Phương trình trên trở thành

$${{x}^{k}}+{{y}^{k}}={{p}^{n}}.$$

Không mất tính tổng quát, giả sử $x\ge y$ . Tương tự bài toán $1$ ta sẽ loại được các trường hợp tầm thường $x=1;x=2$ . Nên ta xét bài toán với trường hợp $x,p\ge 3.$ Do $k$ lẻ, ta có ${{p}^{n}}={{a}^{k}}+{{b}^{k}}=\left( a+b \right)\left( {{a}^{k-1}}-{{a}^{k-2}}b+\cdots-a{{b}^{k-2}}+{{b}^{k-1}} \right)$

Suy ra $p|b+a$. Áp dụng bổ đề số mũ đúng cho $p$ ta có

$${{v}_{p}}\left( {{a}^{k-1}}-{{a}^{k-2}}b+\cdots-a{{b}^{k-2}}+{{b}^{k-1}} \right)={{v}_{p}}\left( {{a}^{k}}+{{b}^{k}} \right)-{{v}_{p}}\left( a+b \right)={{v}_{p}}\left( k \right) $$

Mà ${{a}^{k-1}}-{{a}^{k-2}}b+ \cdots-a{{b}^{k-2}}+{{b}^{k-1}}$ là lũy thừa của $p$ nên ta có $$\left( {{a}^{k-1}}-{{a}^{k-2}}b+\cdots-a{{b}^{k-2}}+{{b}^{k-1}} \right) | k$$

Lập luận tương tự bài toán $1$ ta cũng suy ra vô lí. Vậy phương trình vô nghiệm .

Bài tập 3: Cho số nguyên $n={{2}^{k}},k>1$ , p là số nguyên tố. Chứng minh rằng phương trình ${{a}^{n}}+{{b}^{n}}={{p}^{n}}$ không có nghiệm nguyên dương.

Giải

Tương tự Bài toán $1$, ta loại được các trường hợp tầm thường nên ta chỉ xét đối với trường hợp $a,b$ có ít nhất một số lớn hơn $2$, khi đó $p>3$. Phương trình trở thành dạng

$${{x}^{4}}+{{y}^{4}}={{p}^{{{2}^{k}}}}$$

trong đó $x, y$ có ít nhất một số lớn hơn $2$ và $\left( x,y \right)=1$.

Do $p$ lẻ nên $x, y$ khác tính chẵn lẻ. Không mất tính tổng quát, giả sử $x$ lẻ, $y$ chẵn. Ta có

$${{y}^{4}}={{p}^{{{2}^{k}}}}-{{x}^{4}}=\left( {{p}^{{{2}^{k-1}}}}+{{x}^{2}} \right)\left( {{p}^{{{2}^{k-1}}}}-{{x}^{2}} \right)$$

Do $\left( {{p}^{{{2}^{k-1}}}}+{{x}^{2}};{{p}^{{{2}^{k-1}}}}-{{x}^{2}} \right)=2$ nên

$$\left\{ \begin{array}{l} {{p}^{{{2}^{k-1}}}}+{{x}^{2}}=2{{m}_{1}}^{2} \\ {{p}^{{{2}^{k-1}}}}-{{x}^{2}}=2{{n}_{1}}^{2} \end{array} \right. $$

Suy ra

$$\left\{ \begin{array}{l} {{p}^{{{2}^{k-1}}}}={{m}_{1}}^{2}+{{n}_{1}}^{2} \\ {{x}^{2}}={{m}_{1}}^{2}-{{n}_{1}}^{2} \end{array} \right. $$

và ${{y}^{2}}=2{{m}_{1}}{{n}_{1}}.$

Ta thấy $\left( {{m}_{1}};{{n}_{1}} \right)=1$ vì nếu ngược lại thì ${{m}_{1}}$ và ${{m}_{2}}$ đều phải chia hết cho $p$ (vô lí) nên có các trường hợp sau

$1)$ Nếu $m_1 = m_2^2, n_1=2n_2^2$ và $(m_2,n_2)=1$ thì thế vào ta được

$${{p}^{{{2}^{k-1}}}}={{m}_{2}}^{4}+4{{n}_{2}}^{4}=\left( {{m}_{2}}^{2}+2{{m}_{2}}{{n}_{2}}+2{{n}_{2}}^{2} \right)\left( {{m}_{2}}^{2}-2{{m}_{2}}{{n}_{2}}+2{{n}_{2}}^{2} \right)$$

mà \[\left( {{m}_{2}}^{2}+2{{m}_{2}}{{n}_{2}}+2{{n}_{2}}^{2},{{m}_{2}}^{2}-2{{m}_{2}}{{n}_{2}}+2{{n}_{2}}^{2} \right)=1\] nên \[{{m}_{2}}^{2}-2{{m}_{2}}{{n}_{2}}+2{{n}_{2}}^{2}=1\Leftrightarrow {{\left( {{m}_{2}}-{{n}_{2}} \right)}^{2}}+{{n}_{2}}^{2}=1\Leftrightarrow {{m}_{2}}={{n}_{2}}=1.\] Trường hợp này không thỏa.

$2)$ Nếu $m_1=2m_2^2,n_1=n_2^2$ và $(m_2,n_2)=1$ thì cũng tương tự.

Vậy phương trình không có nghiệm nguyên dương.

Như vậy sử dụng bổ đề số mũ đúng ta đã chứng minh được một trường hợp đặc biệt của Định lí lớn Fermat.

Sau đây, chúng ta sẽ sử dụng Bổ đề số mũ đúng để giải quyết một số bài toán khác.

Bài tập 4: Tìm bộ số nguyên dương $\left( a,b,p \right)$ trong đó $p$ là số nguyên tố thỏa $${{2}^{a}}+{{p}^{b}}={{15}^{a}}.$$

Giải

Ta có $\forall x,y\in \mathbb{Z};n\in \mathbb{N}$ thì ${{x}^{n}}-{{y}^{n}}\vdots x+y$ nên ${{p}^{b}}={{15}^{a}}-{{2}^{a}}\vdots 13\Rightarrow p=13.$

Áp dụng bổ đề

$$b={{v}_{13}}\left( {{13}^{b}} \right)={{v}_{13}}\left( {{15}^{a}}-{{2}^{a}} \right)={{v}_{13}}\left( 15-2 \right)+{{v}_{13}}\left( a \right)\Rightarrow {{v}_{13}}\left( a \right)=b-1\Rightarrow a \ \vdots \  {{13}^{b-1}}$$

Mà $a>0$ nên $a\ge {{13}^{b-1}}$, suy ra

${{13}^{b}}  ={{15}^{a}}-{{2}^{a}}=\left( 15-2 \right)\left( {{15}^{a-1}}+{{15}^{a-2}}.2+\cdots +{{15.2}^{a-2}}+{{2}^{a-1}} \right) $

$ \ge \left( 15-2 \right)\left( {{15}^{{{13}^{b-1}}-1}}+{{15}^{{{13}^{b-1}}-2}}.2+\cdots+{{15.2}^{{{13}^{b-1}}-2}}+{{2}^{{{13}^{b-1}}-1}} \right) $

$\Rightarrow b=1\Rightarrow a=1.$

Vậy nghiệm bài toán là $\left( a,b,p \right)=\left( 1,1,13 \right)$.

 

Bài tập 5: Chứng minh rằng không tồn tại cặp số $\left( a,n \right)$ nguyên dương, $n>2$ , sao cho ${{\left( a+1 \right)}^{n}}-{{a}^{n}}$ là lũy thừa bậc dương của $5.$

Giải

Giả sử tồn tại số nguyên dương $m$ sao cho $${{\left( a+1 \right)}^{n}}-{{a}^{n}}={{5}^{m}}.$$

Nhận xét: nếu$a$ hoặc $a+1$ chia hết cho $5$ thì số còn lại cũng cũng chia hết cho $5$ (vô lí). Nên cả hai số đều không chia hết cho $5.$ Ta xét các trường hợp:

$1.$  Nếu $a\equiv 1\left( \bmod 5 \right)\Rightarrow 0\equiv {{\left( a+1 \right)}^{n}}-{{a}^{n}}\equiv {{2}^{n}}-1\left( \bmod 5 \right)$ . Suy ra $4|n$.

$2.$  Nếu $a\equiv 2\left( \bmod 5 \right)\Rightarrow 0\equiv {{\left( a+1 \right)}^{n}}-{{a}^{n}}\equiv {{3}^{n}}-{{2}^{n}}\left( \bmod 5 \right)$. Suy ra $2|n$.

$3.$  Nếu $a\equiv 3\left( \bmod 5 \right)\Rightarrow 0\equiv {{\left( a+1 \right)}^{n}}-{{a}^{n}}\equiv {{4}^{n}}-{{3}^{n}}\left( \bmod 5 \right)$. Suy ra $4|n$.

Do đó, $n$ luôn là số chẵn, đặt $n=2{{n}_{1}}$, $\left( {{n}_{1}}\in \mathbb{N},{{n}_{1}}\ge 2 \right)$. Ta có

$ {{5}^{m}} = {{\left( a+1 \right)}^{2{{n}_{1}}}}-{{a}^{2{{n}_{1}}}}=\left( {{\left( a+1 \right)}^{2}}-{{a}^{2}} \right)\left( {{\left( a+1 \right)}^{2\left( {{n}_{1}}-1 \right)}}+ \cdots + {{\left( a+1 \right)}^{2}}{{a}^{2\left( {{n}_{1}}-2 \right)}}+{{a}^{2\left( {{n}_{1}}-1 \right)}} \right) $

$=\left( 2a+1 \right)\left( {{\left( a+1 \right)}^{2\left( {{n}_{1}}-1 \right)}}+{{\left( a+1 \right)}^{2\left( {{n}_{1}}-2 \right)}}{{a}^{2}}+…+{{\left( a+1 \right)}^{2}}{{a}^{2\left( {{n}_{1}}-2 \right)}}+{{a}^{2\left( {{n}_{1}}-1 \right)}} \right). $

Suy ra $5| 2a+15$ , áp dụng bổ đề số mũ đúng ta được

${{v}_{5}}\left( {{\left( a+1 \right)}^{2\left( {{n}_{1}}-1 \right)}}+{{\left( a+1 \right)}^{2\left( {{n}_{1}}-2 \right)}}{{a}^{2}}+…+{{\left( a+1 \right)}^{2}}{{a}^{2\left( {{n}_{1}}-2 \right)}}+{{a}^{2\left( {{n}_{1}}-1 \right)}} \right) $

$= {{v}_{5}}\left( {{\left( a+1 \right)}^{2{{n}_{1}}}}-{{a}^{2{{n}_{1}}}} \right)-{{v}_{5}}\left( 2a+1 \right)={{v}_{5}}\left( {{n}_{1}} \right). $

Do ${{\left( a+1 \right)}^{2\left( {{n}_{1}}-1 \right)}}+{{\left( a+1 \right)}^{2\left( {{n}_{1}}-2 \right)}}{{a}^{2}}+ \cdots +{{\left( a+1 \right)}^{2}}{{a}^{2\left( {{n}_{1}}-2 \right)}}+{{a}^{2\left( {{n}_{1}}-1 \right)}}$ là lũy thừa của $5$ nên $${{n}_{1}}\vdots \left( {{\left( a+1 \right)}^{2\left( {{n}_{1}}-1 \right)}}+{{\left( a+1 \right)}^{2\left( {{n}_{1}}-2 \right)}}{{a}^{2}}+…+{{\left( a+1 \right)}^{2}}{{a}^{2\left( {{n}_{1}}-2 \right)}}+{{a}^{2\left( {{n}_{1}}-1 \right)}} \right)$$ (vô lí vì về phải gồm ${{n}_{1}}$ số nguyên dương, ${{n}_{1}}>1$ và $a+1\ge 2$).

Vậy không tồn tại cặp số $\left( a,n \right)$ nguyên dương, $n>2$ sao cho ${{\left( a+1 \right)}^{n}}-{{a}^{n}}$ là lũy thừa bậc dương của $5.$

 

Bài tập 6: Cho hai số nguyên $a,n\ge 2$ sao cho tồn tại số nguyên dương k thỏa $n|{{\left( a-1 \right)}^{k}}$ . Chứng minh rằng n là ước của $1+a+{{a}^{2}}+…+{{a}^{n-1}}$ .

Giải

Giả sử $p$ là ước nguyên tố bất kì của $n$ . Theo giả thiết $n|{{\left( a-1 \right)}^{k}}$ nên p cũng là ước của $a-1$ .

Do ${{a}^{n}}-1=\left( a-1 \right)\left( 1+a+{{a}^{2}}+\cdots +{{a}^{n-1}} \right)$ nên áp dụng bổ đề số mũ đúng ta có

$${{v}_{p}}\left( 1+a+{{a}^{2}}+\cdots+{{a}^{n-1}} \right)={{v}_{p}}\left( {{a}^{n}}-1 \right)-{{v}_{p}}\left( a-1 \right)={{v}_{p}}\left( n \right).$$

Do mọi ước nguyên tố $p$ của n đều thỏa điều trên nên ta có $$n|1+a+{{a}^{2}}+\cdots+{{a}^{n-1}}.$$

Bài tập 7 (HSG Trung Quốc 2009): Tìm cặp số nguyên tố $\left( p,q \right)$ thỏa $pq|{{5}^{p}}+{{5}^{q}}$ (*).

Giải

Ta xét các trường hợp

$1.$   $p=q=5$ thỏa mãn bài toán.

$2.$   Nếu có một số bằng $5$, một số khác $5$. Không mất tính tổng quát, giả sử $p=5;q\ne 5$. Ta có :

$$5q|{{5}^{5}}+{{5}^{q}}\Leftrightarrow q|{{5}^{4}}+{{5}^{q-1}}\Leftrightarrow q|{{5}^{4}}+1=626$$ do ${{5}^{q-1}}\equiv 1\left( \bmod \,q \right)$ nên suy ra $q=2$ hoặc $q=313$.

$3.$  Nếu cả hai số $p,q\ne 5$ . Do ${{5}^{p}}\equiv 5\left( \bmod p \right),\,\,{{5}^{q}}\equiv 5\,\,\,\,\left( \bmod \,q \right)$ nên

$$\left( * \right)\Leftrightarrow \left\{ \begin{array}{l}  {{5}^{p-1}}+1\vdots q \\ {{5}^{q-1}}+1\vdots p \end{array} \right. \Rightarrow \left\{ \begin{array}{l} {{5}^{2\left( p-1 \right)}}-1\vdots q \\ {{5}^{2\left( q-1 \right)}}-1\vdots p \end{array} \right.$$

Do ${{5}^{2\left( p-1 \right)}}-1$ chia hết cho $q$ nhưng ${{5}^{p-1}}-1$ không chia hết cho $q$ nên

$${{v}_{2}}\left( \text{ord}_{q}\left( 5 \right) \right)=1+{{v}_{2}}\left( p-1 \right) .$$

Do ${{5}^{q-1}}-1$ chia hết $q$ nên $q-1\vdots or{{d}_{q}}\left( 5 \right)$ nên

$${{v}_{2}}\left( q-1 \right)\ge 1+{{v}_{2}}\left( p-1 \right) .$$

Tương tự khi xét chia hết cho $p$ ta lại có ${{v}_{2}}\left( p-1 \right)\ge 1+{{v}_{2}}\left( q-1 \right)$ (vô lí).

Vậy các cặp số thỏa mãn là $\left( p,q \right)=\left( 2,5 \right);\left( 5,2 \right);\left( 5,5 \right);\left( 5,313 \right);\left( 313,5 \right).$

Bài tập 8 (HSG Brazil 2009): Cho hai số nguyên tố $p, q$ sao cho $q=2p+1$ . Chứng minh rằng tồn tại một số là bội của $q$ có tổng các chữ số của nó trong hệ cơ số $10$ nhỏ hơn $4.$

Giải

Do $p,q$ đều là số nguyên tố nên $q\ge 5$ .

Nếu $q=5$ thì ta chỉ cần chọn số $10$ thì thỏa yêu cầu bài toán.

Nếu $q>5$ , áp dụng Định lí Fermat nhỏ thì $q|{{10}^{q-1}}-1={{10}^{2p}}-1=\left( {{10}^{p}}-1 \right)\left( {{10}^{p}}+1 \right)$

Suy ra $q|{{10}^{p}}+1$ hoặc $q|{{10}^{p}}-1$.

$1.$  Nếu $q|{{10}^{p}}+1$ thì số $a={{10}^{p}}+1$ là số thỏa yêu cầu đề bài.

$2.$  Nếu $q|{{10}^{p}}-1$. Do $p$ là số nguyên tố và $q$ không là ước của $10-1$(do $q>5$ ) nên $p$ cũng chính là $or{{d}_{q}}\left( 10 \right)$. Do đó $10;{{10}^{2}};\ldots ;{{10}^{p}}$ sẽ có số dư khác nhau khi chia cho $q.$

Ta sẽ có các trường hợp

  • Nếu tồn tại $1\le k\le p$ mà ${{10}^{k}}\equiv p\left( \bmod \,q \right)$ thì ${{2.10}^{k}}+1\equiv 2p+1\equiv 0\left( \bmod \,q \right)$. Khi đó số $a={{2.10}^{k}}+1$ là số thỏa yêu cầu đề bài.
  • Nếu tồn tại $1\le k\le p$ mà ${{10}^{k}}\equiv 2p\left( \bmod \,q \right)$ thì ${{10}^{k}}+1\equiv 2p+1\equiv 0\left( \bmod \,q \right)$. Khi đó số $a={{10}^{k}}+1$ là số thỏa yêu cầu đề bài.
  • Nếu không tồn tại $1\le k\le p$ mà ${{10}^{k}}$ có số dư là $p$ hay $2p$ khi chia cho $q.$ Thì ta sẽ chia các số dư còn lại của $q$ thành $p$ bộ $$\left( 1;2p-1 \right),\left( 2;2p-2 \right),\ldots,\left( p-1;p+1 \right)$$ (tổng $2$ phần tử của một bộ bằng $2p$) . Do tập số dư khi chia cho $q$ của tập $\left\{ 10;{{10}^{2}};\ldots ;{{10}^{p}} \right\}$ có $p$ phần tử nên Theo nguyên lí Dirichlet sẽ có ít nhất hai số ${{10}^{k}}$ và ${{10}^{l}}$ thuộc cùng một bộ. Khi đó số $a={{10}^{k}}+{{10}^{l}}+1$ sẽ chia hết cho $q$ là số thỏa yêu cầu đề bài.

Bài tập 9 (IMO Shortlist 1997): Cho $b,m,n$ là các số nguyên dương thỏa$m>1;\,\,m\ne n$. Biết ${{b}^{m}}-1$và ${{b}^{n}}-1$ có cùng tập hợp các ước nguyên tố. Chứng minh $b+1$ là lũy thừa của $2.$

Giải

Theo đề, gọi $p$ là ước nguyên tố bất kì của ${{b}^{m}}-1$và ${{b}^{n}}-1$.

Ta có kết quả quen thuộc: $$\left( {{b}^{m}}-1,{{b}^{n}}-1 \right)={{b}^{\left( m,n \right)}}-1,$$ đặt $\alpha =\left( m,n \right)$ nên $p|{{b}^{\alpha }}-1$. Suy ra tồn tại $k,l\in \mathbb{N}*$ thỏa $m=\alpha k;\,\,n=\alpha l$.

Đặt $a={{b}^{\alpha }}$ , từ giả thiết suy ra mọi ước nguyên tố của ${{a}^{k}}-1$ và ${{a}^{l}}-1$ đều là ước của $a-1$ . Nói cách khác, tập hợp các ước nguyên tố của ${{a}^{k}}-1,{{a}^{l}}-1$ và $a-1$ là trùng nhau.

Do $m\ne n$ suy ra tồn tại một số $k$ hoặc $l$ lớn hơn 1. Giả sử số đó là k.

Ta chứng minh $a+1$ là lũy thừa của 2.

Thật vậy:

$1.$  Nếu $k$ là số chẵn, đặt $k={{2}^{\beta }}.k’$($k’$ là số lẻ).

Ta có: $${{a}^{k}}-1=\left( {{a}^{k’}}-1 \right)\left( {{a}^{k’}}+1 \right)\left( {{a}^{2k’}}+1 \right)…\left( {{a}^{{{2}^{\beta -1}}k’}}+1 \right).$$

Do đó mọi ước nguyên tố $q$ của ${{a}^{k’}}+1$ cũng là ước của $a-1$

Mà ${{a}^{k’}}+1\vdots a+1$, $\left( a+1;a-1 \right)=1$ hoặc $2.$ Suy ra $2\vdots q\Rightarrow q=2$ nên ${{a}^{k’}}+1$ là lũy thừa của $2.$ Suy ra $a+1$ cũng là lũy thừa của $2.$

$2.$  Nếu $k$ là số lẻ, ta có ${{a}^{k}}-1=\left( a-1 \right)\left( {{a}^{k-1}}+{{a}^{k-2}}+…+a+1 \right)$

Gọi $q$ là ước nguyên tố bất kì của ${{a}^{k-1}}+{{a}^{k-2}}+…+1$. Do ${{a}^{k-1}}+{{a}^{k-2}}+…+a+1$ là số lẻ nên, nên $q$ cũng lẻ và là ước của ${{a}^{k}}-1$ . Do đó q cũng là ước của $a-1$ .

Áp dụng bổ đề số mũ đúng của $q$ ta có

${{v}_{q}}\left( {{a}^{k-1}}+{{a}^{k-2}}+…+1 \right)={{v}_{q}}\left( {{a}^{k}}-1 \right)-{{v}_{q}}\left( a-1 \right)={{v}_{q}}\left( k \right)$

Suy ra $k\vdots \left( {{a}^{k-1}}+{{a}^{k-2}}+…+1 \right)$ (vô lí vì vế phải có k số nguyên dương, $a>1$ ).

Vậy $a+1={{b}^{\alpha }}+1$ là lũy thừa của $2$.

Vì ${{b}^{\alpha }}+1$ là lũy thừa của $2$ nên nếu $\alpha $ là số chẵn thì ${{b}^{\alpha }}+1={{\left( {{b}^{\alpha ‘}} \right)}^{2}}+1$ hoặc là số lẻ hoặc chia 4 dư 2 nên chỉ có một trường hợp thỏa là $b=1$ . Còn nếu $\alpha $ là số lẻ thì ${{b}^{\alpha }}+1=\left( b+1 \right)\left( {{b}^{\alpha -1}}+{{b}^{\alpha -2}}+…+b+1 \right)$ nên $b+1$ cũng là lũy thừa của $2$.

Bài tập 10 (IMO Shortlist 1999): Tìm các số nguyên dương $n,p$ trong đó p nguyên tố thỏa ${{n}^{p-1}}|{{\left( p-1 \right)}^{n}}+1$.

Giải

Ta xét các trường hợp sau

$1.$  Nếu $p=2\Rightarrow n|2\Rightarrow n=1;2$ (thỏa).

$2.$  Nếu $p>2$ , suy ra $p$ lẻ nên ${{\left( p-1 \right)}^{n}}+1$ lẻ $\Rightarrow n$ lẻ

Gọi $q$ là ước nguyên tố nhỏ nhất của n $\Rightarrow q|{{n}^{p-1}}|{{\left( p-1 \right)}^{n}}+1$ $\Rightarrow q|{{\left( p-1 \right)}^{2n}}-1$

Mà : $q|{{\left( p-1 \right)}^{q-1}}-1\Rightarrow q|{{\left( p-1 \right)}^{\left( 2n,q-1 \right)}}-1$

Do n lẻ và $q$ là ước nguyên tố nhỏ nhất của n nên $\left( 2n;q-1 \right)=2$ .

Suy ra $q|{{\left( p-1 \right)}^{2}}-1=\left( p-2 \right)p$ $\Rightarrow $ $q|p-2$ hoặc $q=p$. Ta lại có các trường hợp nhỏ

$(a)$  Nếu $q|p-2\Rightarrow 0\equiv {{\left( p-1 \right)}^{n}}+1\equiv 1+1\equiv 2\left( \bmod \,q \right)$ $\Rightarrow q=2$ (vô lí vì q lẻ)

$(b)$  Nếu $q=p$ . Áp dụng bổ đề số mũ đúng cơ số q ta có

$\left( p-1 \right){{v}_{p}}\left( n \right)={{v}_{p}}\left( {{n}^{p-1}} \right)\le {{v}_{p}}\left[ {{\left( p-1 \right)}^{n}}+1 \right]={{v}_{p}}\left( p-1+1 \right)+{{v}_{p}}\left( n \right)=1+{{v}_{p}}\left( n \right)$

Suy ra : $\left( p-2 \right){{v}_{p}}\left( n \right)\le 1\Rightarrow p=3$ và ${{v}_{p}}\left( n \right)=1.$

Đến đây, bài toán trở thành : Tìm n để ${{n}^{2}}|{{2}^{n}}+1$.

Nhận xét $n=1$ thỏa yêu cầu bài toán nên ta xét $n>1$. Suy ra $n$ là số lẻ, gọi $r$ là ước nguyên tố nhỏ nhất của $n$. Suy ra $r|{{2}^{n}}+1\,\,|{{2}^{2n}}-1$, mà $r|{{2}^{r-1}}-1$ nên suy ra $r|{{2}^{\left( 2n;r-1 \right)}}-1$.

Do $n$ là số lẻ và $r$ là ước nguyên tố nhỏ nhất của $n$ nên $\left( 2n;r-1 \right)=2$ nên $r=3$. Ta có đánh giá sau

$$2{{v}_{3}}\left( n \right)\le {{v}_{3}}\left( {{4}^{n}}-1 \right)={{v}_{3}}\left( 4-1 \right)+{{v}_{3}}\left( n \right)\Rightarrow {{v}_{3}}\left( n \right)\le 1\Rightarrow {{v}_{3}}\left( n \right)=1.$$ Suy ra $n=3.m$, $\left( m,n \right)=1$. Thế vào đề bài, ta được $${{m}^{2}}|{{8}^{m}}+1|{{8}^{2m}}-1.$$

Nếu $m>1$ , tương tự ta gọi $s$ là ước nguyên tố nhỏ nhất của $m.$ Suy ra $m$ là ước của ${{8}^{2}}-1=63$. Do đó $s=7$, điều này vô lí vì ${{8}^{m}}+1$ chia $7$ dư $2.$ Suy ra $m=1\Rightarrow n=3$.

Vậy $\left( n,p \right)=\left( 1,2 \right);\left( 2,2 \right);\left( 3;3 \right)$ .

Một số lưu ý chuẩn bị cho kì thi vào lớp 10: Toán chung

Năm nay TPHCM và PTNK thi vào lớp 10 ba môn chung: Toán, Văn, Anh. Cũng sắp tới ngày thi, giai đoạn này cần tập trung vào việc học tập, ôn luyện rèn luyện giải đề…để có một kì thi thành công, kết quả như ý. Nhân đây tôi cũng có một số điều muốn chia sẻ trong giai đoạn nước rút này.

Đại số

  • Ôn tập rút gọn các biểu thức, chú ý các hằng đẳng thức, chú ý sai dấu.
  • Phương trình: Xem lại các giải pt vô tỷ, điều kiện, phương pháp giải, phương trình tích. Hệ phương trình xe, kĩ phương pháp thế, cộng đại số, ẩn phụ.
  • Viete chú ý các xử lí biểu thức chứa biết đối xứng hay không đối xứng, điều kiện có nghiệm.

Hình học

  • Nắm chắc hệ thức lượng, tỉ số lượng giác, công thức diện tích, chú ý các bài tính toán độ dài.
  • Hình học chú ý các các tính chất tiếp tuyến, phương pháp chứng minh tiếp tuyến, tính chất 2 tiếp tuyến cắt nhau và các bài toán liên quan.
  • Phương pháp chứng minh tứ giác nội tiếp,  các loại góc, các tính chất quen thuộc.

Toán thực tế

  • Chú ý các bài toán về phần trăm, giá cả, năng suất.
  • Hỏi cái nào, đặt ẩn cái đó, tìm mối tương quan giữa các đại lượng để lập phương trình hay hệ phương trình.
  • Nắm chắc các kĩ thuật giải pt, hpt, chú ý điều kiện của  ẩn.
  • Chú ý các công thức tính chu vi, diện tích, thể tích các hình quen thuộc.

Chúc các em có mùa thi thành công!

Các bài toán tổ hợp trên dãy số

CÁC BÀI TOÁN TỔ HỢP TRÊN DÃY SỐ

Thầy Lê Phúc Lữ 

(Lớp Cao học Khoa học tự nhiên TP.HCM)

Trong bài viết nhỏ này, chúng ta sẽ cùng xét khía cạnh tổ hợp của dãy số nguyên; khi cần đếm số lượng dãy thỏa mãn một điều kiện cho trước nào đó. Các phương pháp thường gặp: truy hồi, xuống thang, cực hạn, phản chứng, …

1. Các bài toán chọn lọc

Bài tập 1.1: Tìm tất cả các bộ số nguyên dương $x_1,\ x_2,\ x_3,\ \ldots ,\ x_{2017}$ sao cho có thể đặt chúng lên vòng tròn theo thứ tự đó mà $6$ số liên tiếp bất kỳ đều có thể chia thành hai nhóm $3$ có tổng bằng nhau.

Giải

Dùng phương pháp xuống thang.

Ta có $x_i+x_{i+1}+x_{i+2}+x_{i+3}+x_{i+4}+x_{i+5} \equiv 0 \pmod{2}$ với mọi $i=1,2,3,\ldots ,2017$ nên $x_i \equiv x_{i+6}$ với mọi $i.$

Vì $(6,2017)=1$ nên suy ra tất cả các số có cùng tính chẵn lẻ. Ta xét phép biến đổi dãy số sau:

  • Nếu tất cả các số cùng chẵn thì thay bằng $y_i=\dfrac{x_i}{2}$.
  • Nếu tất cả các số cùng lẻ thì thay bằng $y_i=\dfrac{x_i+1}{2}$.

Dễ thấy dãy mới cũng thỏa và tổng $S=\sum\limits_{i=1}^{2017} a_i$ sẽ giảm ngặt nếu có một số nào đó trong dãy khác $1$; suy ra quá trình biến đổi sẽ dừng lại khi tất cả đều là $1$. Vì ta thu được một dãy toàn là $1$ nên dãy ban đầu có tất cả các số hạng bằng nhau.

Nhận xét: Bài toán trên có thể thay việc chia 2 nhóm thành $3,4,5,\ldots $ nhóm và vẫn giải được bằng cách tương tự. Ta xét các bài tương tự sau:

Bài tập 1.2 (APMO 2017): Bộ năm số nguyên là tốt nếu có thể đặt chúng là $a,b,c,d,e$ để $a-b+c-d+e=29.$ Tìm tất cả các bộ $2017$ số sao cho $5$ số liên tiếp bất kỳ trong chúng đều tốt.

Ở bài toán này, điểm khó là không biết các số đã cho có dương hay không; vì thể, đại lượng tổng ở trên không xét tiếp tục được.

Tuy nhiên, cách áp dụng vẫn tương tự như sau:

  • Trừ tất cả các số của bộ cho $29$, ta thu được điều kiện tốt trở thành $a-b+c-d+e=0.$
  • Tất cả các số đã cho cùng tính chẵn lẻ, và chính xác là cùng chẵn.
  • Xét đại lượng $S=\sum\limits_{i=1}^{2017}{\left| \frac{{{a}_{i}}}{2} \right|}$ thì thông qua phép chia 2, tổng này giảm ngặt. Từ đó suy ra tất cả các số này phải là $0$ và tất cả ban đầu phải là $29.$

Bài tập 1.3 (VMO 2014): Tìm tất cả các bộ số $2014$ số hữu tỷ không âm sao cho nếu bỏ đi bất kỳ số nào trong chúng thì các số còn lại có thể được chia thành $3$ nhóm rời nhau, mỗi nhóm có $671$ số sao cho tích các số trong mỗi nhóm là bằng nhau.

Bài này khó hơn vì: số hữu tỷ chứ không nguyên, tích chứ không phải tổng, … Ta lần lượt giải quyết điều đó như sau:

  • Quy đồng mẫu để đưa về số nguyên.
  • Xét số mũ của 1 ước nguyên tố để đưa về tổng.
  • Chú ý thêm trường hợp số 0 (nếu có 1 số thì phải có ít nhất 4 số).

Bài tập 1.4: Cho dãy số nguyên dương $({{a}_{n}})$ thỏa mãn:

$i)$ Gồm các số phân biệt nhau.

$ii)$ Với mọi $n$ thì ${{a}_{n}}\ge n.$

$iii)$ $a_1=5,\ a_2=4,\ a_3=3$.

a) Chứng minh rằng tồn tại $n>2017$ sao cho $a_n \ne n+1$?

b) Giả sử $a_n=n+2$ với mọi $n>2017$, hỏi có tất cả bao nhiêu dãy số như thế?

Giải

a) Bài toán có thể giải quyết dễ dàng bằng phản chứng và Dirichlet. Thật vậy, nếu ${{a}_{n}}=n+1$ với mọi $n>2017$ thì các số hạng ${{a}_{4}}\to {{a}_{2017}}$ sẽ nhận các giá trị trong tập hợp $6\to 2018$. Khi đó, sẽ có hai số hạng bằng nhau, không thỏa.

b) Nếu đã có ${{a}_{n}}=n+2$ với mọi $n>2017$ thì các số hạng ${{a}_{4}}\to {{a}_{2017}}$ sẽ nhận các giá trị trong tập hợp $6\to 2019.$ Nhận xét:

  • ${{a}_{2017}}\in \left\{ 2017,2018,2019 \right\}$ nên có $3$ cách chọn.
  • ${{a}_{2016}}\in \left\{ 2016,2017,2018,2019 \right\}$ nhưng vì ${{a}_{2017}}$ đã lấy một số nên cũng còn $3$ cách chọn.
  • Tương tự, đến ${{a}_{6}}$ vẫn có $3$ cách chọn. Còn lại ${{a}_{5}}$ có $2$ cách chọn và ${{a}_{4}}$ có $1$ cách chọn.

Theo nguyên lý nhân, ta có $2\cdot {{3}^{2012}}$ dãy thỏa mãn.

Bài tập 1.5: Xét lục giác $ABCDEF$ có độ dài cạnh là $1$ được điền các số như hình vẽ

Một con ếch xuất phát từ $A$ và nhảy đến các đỉnh sao cho mỗi bước nhảy đều có độ dài nguyên. Hành trình của ếch là dãy các tên đỉnh mà ếch đã nhảy qua; và hai hành trình được coi là khác nhau nếu ở một lần thứ $k$ nào đó, đỉnh mà ếch nhảy đến ở hai hành trình là khác nhau.

Gọi $m$ là số hành trình ếch nhảy sao cho tổng các số mà nó nhảy qua là $2017$. Chứng minh rằng $m$ không phải là số chính phương.

Giải

Ta thấy $ACE$ và $BDF$ là hai tam giác đều có cạnh là $\sqrt{3}$ nên mỗi lần, ếch sẽ nhảy từ tam giác đều này đến tam giác đều kia.

Chia nhóm:

  • $I=(A,C,E)$ tương ứng với các số $(0,0,1)$.
  • $II=(B,D,F)$ tương ứng với $(1,1,2)$.

Ta thấy $\left\{ x+y|x\in I,y\in II \right\}=\left\{ 1,1,1,1,2,2,2,2,3 \right\}$ chứng tỏ tổng các số trên hai bước nhảy liên tiếp của ếch sẽ nhận giá trị là $4$ số $1$, $4$ số $2$ và $1$ số $3.$ Nếu gọi ${{s}_{n}}$ là số hành trình của ếch có tổng là $n$ thông qua chẵn bước thì

$${{s}_{n}}=4{{s}_{n-1}}+4{{s}_{n-2}}+{{s}_{n-3}}.$$

Một cách tương tự, gọi ${{t}_{n}}$ là số hành trình của ếch có tổng là $n$ thông qua lẻ bước thì công thức truy hồi vẫn thế (chỉ khác ở các số hạng đầu).

Vì vậy nên nếu gọi ${{u}_{n}}={{s}_{n}}+{{t}_{n}}$ là số hành trình của ếch có tổng là $n$ thì

$${{u}_{n}}=4{{u}_{n-1}}+4{{u}_{n-2}}+{{u}_{n-3}} \text{ với } n\ge 3.$$

Ta có ${{u}_{0}}=1,{{u}_{1}}=6,{{u}_{2}}=28$ và từ công thức truy hồi thì $m={{u}_{2017}}\equiv {{u}_{1}}\equiv 2 \pmod{4}$ nên $m$ không thể là số chính phương, ta có đpcm.

Nhận xét: Bài toán có thể giải bằng cách gọi $6$ dãy truy hồi $a_n,\ b_n,\ c_n,\ d_n,\ e_n,\ f_n$ chỉ số hành trình của ếch có tổng là $n$ và kết thúc tại $A,B,C,D,E,F$. Tuy nhiên, cách tiếp cận đó khá phức tạp, đòi hỏi phải khai thác nhiều các liên hệ giữa các đường đi.

Một bài toán tương tự:

Bài tập 1.6 (Ả Rập TST 2017): Người ta đặt các số $1,2,3,4$ trên vòng tròn theo thứ tự đó. Một con kiến xuất phát từ số $1$ và ở mỗi bước, nó sẽ bò qua số bên cạnh. Hỏi con kiến có bao nhiêu cách bò sao cho tổng tất cả các số mà nó bò qua (kể cả số ban đầu) bằng 21?

Tương tự bài trên, ta cũng tìm được hệ thức truy hồi là $s_n=s_{n-3}+2s_{n-5}+s_{n-7}$. Từ đó tính được $s_{21}=167.$

Bài tập 1.7: Đếm số dãy số nguyên dương $\left( a_1,\ a_2,\ \ldots ,\ a_{12}\right) $ thỏa mãn các điều kiện sau:

a) $1\le a_1 \le a_2 \le \ldots \le a_{12} \le 2017$

b) $a_i \equiv i^2 (\bmod 12)$.

Giải

Theo giả thiết, ta có

${{a}_{1}}\equiv {{a}_{5}}\equiv {{a}_{7}}\equiv {{a}_{11}}\equiv 1\text{ }(\bmod 12) $

$ {{a}_{2}}\equiv {{a}_{4}}\equiv {{a}_{8}}\equiv {{a}_{10}}\equiv 4\text{ }(\bmod 12) $

$ {{a}_{3}}\equiv {{a}_{9}}\equiv 9\text{ }(\bmod 12) $

$ {{a}_{6}}\equiv {{a}_{12}}\equiv 0\text{ }(\bmod 12) $

Đặt ${{a}_{i}}=12{{b}_{i}}+{{r}_{i}}$ với $i=1,2,3,\ldots ,12$ và ${{r}_{i}}$ là số dư tương ứng đã chỉ ra ở trên.

Do tính không giảm của dãy nên ta phải có

$$0\le {{b}_{1}}\le {{b}_{2}}\le {{b}_{3}}<{{b}_{4}}<{{b}_{5}}<{{b}_{6}}\le {{b}_{7}}\le {{b}_{8}}\le {{b}_{9}}<{{b}_{10}}<{{b}_{11}}<{{b}_{12}}\le 168.$$

Từ đó suy ra

$0\le {{b}_{1}}<{{b}_{2}}+1<{{b}_{3}}+2<{{b}_{4}}+2<{{b}_{5}}+2<{{b}_{6}}+2\le {{b}_{7}}+3\le {{b}_{8}}+4\le {{b}_{9}}+5 $

$<{{b}_{10}}+5<{{b}_{11}}+5<{{b}_{12}}+5\le 173 $

Do các số liệt kê ở trên đều phân biệt và thuộc $[0;173]$ nên số cách chọn một bộ như thế là $C_{174}^{12}$. Đó cũng chính là số dãy cần tìm.

Nhận xét: Điều kiện thứ hai có thể thay bằng một hàm số tùy ý theo $i$ chứ không nhất thiết phải là ${{i}^{2}}$, cách giải vẫn tương tự như trên.

Bài tập 1.8: Hỏi có bao nhiêu hoán vị $a_1,\ a_2,\ …,\ a_{2017}$ của $2017$ số nguyên dương đầu tiên thỏa mãn đồng thời các điều kiện sau:

$i)$ $a_{i+1}-a_i\le 1$ với mọi $i=1,2,3,\ldots,2016.$

$ii)$ Có đúng một chỉ số $i$ với $1\le i\le 2017$ sao cho $a_i=i$?

Giải

Trước hết, ta sẽ chứng minh nhận xét rằng số hoán vị của $n$ số nguyên dương đầu tiên thỏa mãn điều kiện i), gọi là hoán vị đẹp, sẽ là ${{2}^{n-1}}$. Thật vậy,

  • Đầu tiên, ta đặt số $1$ vào hoán vị.
  • Số $2$ có thể xếp trước hoặc sau số $1$, có $2$ cách.
  • Số $3$ có thể xếp vào đầu dãy hoặc ngay sau số $2$ đã xếp trước đó, có $2$ cách.
  • Số $4$ có thể xếp vào đầu dãy hoặc ngay sau số $3$ đã xếp trước đó, cũng có $2$ cách. Cứ như thế cho đến $n.$

Do đó, có tất cả ${{2}^{n-1}}$ cách xếp, tương ứng vời ${{2}^{n-1}}$ hoán vị.

Tiếp theo, giả sử ta có ${{a}_{i}}=i$.

Khi đó ${{a}_{i+1}}\le {{a}_{i}}+1=i+1$, nhưng không thể có ${{a}_{i+1}}=i+1$ (do chỉ có 1 chỉ số thỏa mãn ii) nên ${{a}_{i+1}}\le i$, mà ${{a}_{i}}=i$ nên ${{a}_{i+1}}\le i-1$. Tiếp theo, ${{a}_{i+2}}\le {{a}_{i+1}}+1\le i$ nên ${{a}_{i+2}}\le i-1$.

Do đó, các số từ ${{a}_{i+1}}$ đến ${{a}_{2017}}$ nhận giá trị không vượt quá $i-1$.

Lập luận tương tự, các số từ ${{a}_{1}}$ đến ${{a}_{i-1}}$ phải nhận giá trị không nhỏ hơn $i+1.$

Do đó, hai đoạn hoán vị phía trước và phía sau ${{a}_{i}}$ phải có độ dài bằng nhau, tức là ${{a}_{1009}}=1009$ là số ở giữa.

Rõ ràng các hoán vị phía trước và phía sau $1009$ đều phải là hoán vị đẹp và được sắp xếp độc lập với nhau.

Vậy số hoán vị cần tìm là ${{\left( {{2}^{1007}} \right)}^{2}}={{2}^{2014}}.$

Nhận xét: Nếu đề đổi số $2017$ thành $2018$ thì sẽ tồn tại hai chỉ số $i$ như trên và chúng sẽ cách đều hai đầu $1$ và $2018$. Khi đó, đoạn ở giữa cũng sẽ cố định, tức là có $i<j$ để

${{a}_{k}}=k$ với mọi $k=i,i+1,\ldots ,j$ và $i+j=2019.$

Phần trước $i$ và phần sau $j$ sẽ đổi chỗ cho nhau với số cách xếp là ${{({{2}^{i-1}})}^{2}}$.

Bài tập 1.9: Cho dãy các số nguyên dương $(u_n)$ thỏa mãn điều kiện

$0\le u_{m+n}-u_m-u_n \le 1$ với mọi $m,n\in \mathbb{Z}^+$.

Chứng minh rằng tồn tại $a\in \mathbb{R}^+$ sao cho $-1\le u_n-\left[ an \right]\le 1$ với mọi $n=1,2,3,\ldots ,2017.$

Giải

Ta đưa điều cần chứng minh về

$$\frac{{{u}_{n}}}{n}<a<\frac{{{u}_{n}}+1}{n}.$$

Đến đây, gọi

$$m=\min \left\{ \left. \frac{{{u}_{n}}+1}{n} \right|n=1,2,3,\ldots ,2017 \right\}$$ và

$$M=\max \left\{ \left. \frac{{{u}_{n}}}{n} \right|n=1,2,3,\ldots ,2017. \right\}$$

Cần chỉ ra $m>M$ rồi chọn số $a$ nằm giữa $(m,M)$ là xong. Gọi $p,q$ lần lượt là các chỉ số nhỏ nhất để có dấu bằng xảy ra ở các đánh giá trên. Khi đó

${{u}_{p}}+1=pm$ và ${{u}_{q}}=Mq.$

Ngoài ra, ${{u}_{k}}+1>km,\forall k<p$ và ${{u}_{k}}<kq,\forall k<q.$

  • Nếu $p=q$ thì hiển nhiên đúng.
  • Nếu $p>q$, ta đặt $p=q+k$ thì $k<p$ nên ${{u}_{k}}+1>km$, vì ${{u}_{p}}\ge {{u}_{q}}+{{u}_{k}}$ (theo giả thiết) nên $pm-1>Mq+km-1\Leftrightarrow m>M.$
  • Nếu $p<q$ thì cũng chứng minh tương tự với chú ý rằng ${{u}_{q}}\le {{u}_{p}}+{{u}_{k}}+1.$

Nhận xét: Nếu đề bài đổi giả thiết thành $0\le u_{m+n}-u_m-u_n\le 2$,

ta sẽ cần đến hai số $a,b$ sao mới thỏa mãn được kết luận (vì khoảng chênh lệch của các số hạng rộng hơn một tí), cụ thể là tồn tại $a,b>0$ để

$$-1\le u_n-\left[ an \right]-\left[ bn \right]\le 1.$$

Ở bài toán trên, ta còn chứng minh được một kết quả mạnh hơn là tồn tại $a$ để $u_n=[an]$ với mọi $n.$ Một bài toán tương tự trong đề trường Đông miền Trung:

Bài tập 1.10: Cho hàm số $f:\mathbb{R}\to \mathbb{R}$ thỏa mãn $\left| f(x+y)-f(x)-f(y) \right|\le 1,\forall x,y\in \mathbb{R}$. Chứng minh rằng tồn tại hàm cộng tính $g:\mathbb{R}\to \mathbb{R}$ thỏa mãn $\left| f(x)-g(x) \right|\le 1,\forall x.$

Đây có thể nói là một phiên bản trên $\mathbb{R}$ của bài toán trên (thay vì xét trên $\mathbb{N}$).

Tiếp theo, ta xét lớp các bài toán sử dụng một định lý thú vị trong dãy số, số học. Trước hết, ta xét định lý Beatty với nội dung như sau:

Cho hai số vô tỷ dương $\alpha ,\beta $. Xét hai dãy số:

  • $[\alpha ],[2\alpha ],[3\alpha ],\ldots $ tạo thành dãy $A.$
  • $[\beta ],[2\beta ],[3\beta ],\ldots $ tạo thành dãy $B.$

Khi đó $\dfrac{1}{\alpha }+\dfrac{1}{\beta }=1$ khi và chỉ khi $A,B$ là phân hoạch của $\mathbb{Z}^+$.

Chứng minh

Định lý này có thể chứng minh bằng cách sử dụng các BĐT về phần nguyên. Dưới đây là cách chứng minh cho chiều đảo:

Với mỗi số nguyên dương $k$, gọi $m,n$ là các số nguyên dương thỏa mãn

$$[m\alpha ]\le k<[(m+1)\alpha ] \text{ và } [n\beta ]\le k<[(n+1)\beta ].$$

Đặt $A=\{[i\alpha ],1\le i\le m\}$ và $B=\{[j\beta ],1\le j\le n\}$ thì $\left| A \right|=m,\left| B \right|=n$ và $A,B$ là phân hoạch của tập hợp $\left\{ 1,2,3,\ldots ,k \right\}$ theo định nghĩa của đề bài.

Do đó $m+n=k$. Theo bất đẳng thức phần nguyên thì $m\alpha -1<k<(m+1)\alpha $ nên $\dfrac{m}{k+1}<\dfrac{1}{\alpha }<\dfrac{m+1}{k}$.

Tương tự $\dfrac{n}{k+1}<\dfrac{1}{\beta }<\dfrac{n+1}{k}.$

Suy ra $$\dfrac{m+n}{k+1}<\dfrac{1}{\alpha }+\dfrac{1}{\beta }<\dfrac{m+n+2}{k} \text{ hay } \dfrac{k}{k+1}<\dfrac{1}{\alpha}+\dfrac{1}{\beta }<\dfrac{k+2}{k}.$$

Cho $k\to +\infty $, ta thu được $\dfrac{1}{\alpha }+\dfrac{1}{\beta }=1.$

Bài tập 1.11: Hai dung dịch $A,B$ có đặc điểm: số đo thể tích của $1$ kg $A$ bằng số đo khối lượng của $1$ lít $B.$ Ngoài ra, $p$ lít $A$ nặng bằng $q$ lít $B$ với $p,q$ nguyên tố khác nhau. Mỗi dung dịch được chia cho vào các bình nhỏ giống nhau, cùng chứa $1$ lít và vỏ nặng $1$ kg. Chứng minh rằng có đúng một cách ghép các bình cùng loại ($A$ hoặc $B$) lại với nhau mà khối lượng của chúng thuộc khoảng $(2017;2018).$

Giải

Gọi $x,y$ lần lượt là khối lượng riêng của các dung dịch thì $\dfrac{1}{x}=1\cdot y,px=qy$ nên $x=\sqrt{\dfrac{q}{p}},y=\sqrt{\dfrac{p}{q}}.$

Khối lượng mỗi bình là $\alpha =1+\sqrt{\dfrac{q}{p}},\beta =1+\sqrt{\dfrac{p}{q}}$. Dễ thấy $\dfrac{1}{\alpha }+\dfrac{1}{\beta }=1$, thỏa mãn định lý Beatty.

Suy ra hai dãy $[m\alpha ],[n\beta ]$ là phân hoạch của số nguyên dương nên ta có đpcm.

Bài tập 1.12 (APMO 2006): Với mỗi số nguyên dương $n$, gọi $a_n,\ b_n$ lần lượt là số cách viết $10^n$ trong hệ nhị phân, ngũ phân. Chứng minh rằng $(a_n),(b_n)$ là phân hoạch của $\mathbb{Z}^+ \backslash \{1\}.$

Giải

Để giải bài này, chú ý rằng: số chữ số của $M$ trong hệ $p$ phân là $[{{\log }_{p}}M]+1$.

Ngoài ra, $\alpha ={{\log }_{2}}10,\beta ={{\log }_{5}}10$ thỏa mãn điều kiện của định lý Beatty.

Từ đó, ta có một nhận xét thú vị rằng: tổng số chữ số của ${{2}^{n}}$ và ${{5}^{n}}$ trong hệ thập phân là $n+1.$

Bài tập 1.13 (VN TST 2000): Cho số nguyên dương $k$. Dãy số $(u_n)$ xác định bởi: $u_1=1$ và $u_{n+1}$ là số nguyên dương nhỏ nhất không thuộc tập hợp

$$\left\{ u_1,\ u_2,\ \ldots ,\ u_n,\ u_1+k,\ u_2+2k,\ \ldots ,\ u_n+nk \right\}.$$

Chứng minh rằng tồn tại $\alpha $ vô tỷ dương sao cho $u_n=\left[ n\alpha \right]$ với mọi $n.$

Giải

Để giải bài toán này, ta xét đa thức $P(x)={{x}^{2}}+(k-2)x-k$ với $k$ là số nguyên dương đã cho thì $P(x)$ có hai nghiệm phân biệt trái dấu. Hơn nữa, ${{\Delta }_{P(x)}}={{(k-2)}^{2}}+4k={{k}^{2}}+4$, không thể là số chính phương với bất kì số k nguyên dương nào nên hai nghiệm này đều là số vô tỉ. Ta thấy $$P(1)=1+(k-2)-k=-1<0,P(2)=4+2(k-2)-k=k>0$$ nên nghiệm dương của phương trình $P(x)=0$ thuộc khoảng $(1,2)$. Gọi nghiệm đó là $a.$

Đặt $b=a+k$ thì $a,b$ đều vô tỉ và $ab=a(a+k)={{a}^{2}}+ak=2a+k=a+b$ nên $\dfrac{1}{a}+\dfrac{1}{b}=1$.

Xét $f(n)=[na],g(n)=[nb]=f(n)+kn$ với $n$ là số nguyên dương.

Ta sẽ chứng minh rằng ${{x}_{n}}=f(n)$ bằng quy nạp. Thật vậy,

– Với $n=1$, khẳng định hiển nhiên đúng vì $1<a<2.$

– Giả sử ${{x}_{n}}=f(n)$ với mọi $n=1,2,3,…,m$. Ta sẽ chứng minh rằng ${{x}_{m+1}}=f({{x}_{m+1}})$.

Ta có $f(i)={{x}_{i}},g(i)=f(i)+ik={{x}_{i}}+ik$ với mọi $i=1,2,3,…,m$ nên ta có tập hợp

$H=\left\{ {{x}_{1}},{{x}_{2}},…,{{x}_{m}},{{x}_{1}}+k,{{x}_{2}}+2k,…,{{x}_{m}}+mk \right\} $

$ =\left\{ f(1),f(2),…,f(m),g(1),g(2),…,g(m) \right\} $

Rõ ràng $f(m+1)\notin H$ và $g(n)>f(n)$ với mọi $n$, $f(n)$ là hàm số đồng biến trên $\mathbb{N}*$ nên ta thấy rằng $f(m+1)$ chính là số tự nhiên nhỏ nhất không thuộc H. Theo định nghĩa dãy số $({{x}_{n}})$ đã cho thì ta có ${{x}_{m+1}}=f(m+1)$.

Do đó, khẳng định cũng đúng với $m+1.$ Theo nguyên lí quy nạp, ta có đpcm. Vậy số tự nhiên cần tìm chính là $a$ là nghiệm dương của phương trình ${{x}^{2}}+(k-2)x-k=0$.

Nhận xét: Đây là một kết quả có từ $1959$. Ta có thể phân tích cách tiếp cận như sau:

Xuất phát từ việc $\alpha =\sqrt{2},\beta =\sqrt{2}+2$ thỏa mãn điều kiện Beatty. Ta có hai dãy với công thức

$a_n=\left[ n\sqrt{2} \right],\ b_n=a_n+2n$ là phân hoạch của $\mathbb{Z}^+$.

Từ đó, để giấu dãy $b_n$ đi, ta chỉ cần xét $a_n+2n$.

Để ý $a_1=1,\ a_2=2,\ a_3=4,\ b_1=3,\ b_2=6,\ b_3=10$ nên $a_4$ có thể định nghĩa là số nguyên dương nhỏ nhất không thuộc $\left\{ a_1,\ a_2,\ a_3,\ a_1+2,\ a_2+4,\ a_3+6 \right\}$. Đó chính là cơ sở để có bài toán trên.

Bài tập 1.14 (Dãy Wythoff): Cho chuỗi $S_1=1$. Chuỗi $S_n$ được tạo thành từ chuỗi $S_{n-1}$ bằng cách thay $1\to 01$ và $0\to 1.$ Các chuỗi $S_1,\ S_2,\ S_3,\ \ldots $ được ghép liên tiếp lại với nhau thành một chuỗi vô hạn $L$. Gọi $a_n$ là vị trí của số $1$ thứ $n$ trong chuỗi $L.$ Chứng minh rằng tồn tại $\alpha $ vô tỷ dương sao cho $a_n=\left[ n\alpha \right],\forall n.$

Ở đây, ta có nhận xét rằng số $0$ thứ $n$ được sinh ra bởi số $1$ thứ $n$ nên nếu gọi $k_n$ là số các số $0$ đứng trước số $1$ thứ $n$ và $b_n$ là vị trí của số $0$ thứ $n,$ ta sẽ có $a_n=n+k_n$ và $b_n=2n+k_n$ nên $b_n=a_n+n$.

Chú ý rằng $(a_n),\ (b_n)$ chính là phân hoạch của $\mathbb{Z}^+$ nên dễ dàng tìm được $\alpha $ là nghiệm của $\dfrac{1}{\alpha }+\dfrac{1}{\alpha +1}=1$ hay $\alpha $ chính là tỷ số vàng.

Bài tập 1.15: Cho $n$ là số nguyên dương, hỏi có bao nhiêu dãy số $a_1,\ a_2,\ \ldots ,\ a_{2n}$ sao cho

$i)$ $a_i \in \left\{ -1,1 \right\}$ với $i=1,2,3,\ldots ,2n.$

$ii)$ $\left| \sum\limits_{i=2k+1}^{2l} a_i \right|\le 2$ với $0\le k<l\le n$?

Giải

Gọi $S$ là tập hợp các dãy thỏa mãn đề bài và đặt $\left| S \right|={{s}_{n}}$. Gọi $T$ là tập hợp tất cả các tổng các ${{a}_{i}}$ lấy từ chỉ số lẻ bất kỳ đến $2n.$ Theo giả thiết thì $T\subset \left\{ \pm 2,\pm 1,0 \right\}$, tuy nhiên, tất cả các tổng trong $T$ đều có chẵn số hạng mà mỗi số hạng đều là $\pm 1$ nên tất cả phải đều chẵn. Suy ra $T\subset \left\{ \pm 2,0 \right\}$.

Nếu trong $T$ chứa cả $2$ lẫn $-2$ thì giả sử $\sum\limits_{k=2i+1}^{2n}{{{a}_{k}}}=2$ và $\sum\limits_{k=2j+1}^{2n}{{{a}_{k}}}=-2$ với $i<j$ , khi đó

\[4=\sum\limits_{k=2i+1}^{2n}{{{a}_{k}}}-\sum\limits_{k=2j+1}^{2n}{{{a}_{k}}}=\sum\limits_{k=2i+1}^{2j}{{{a}_{k}}},\] mâu thuẫn.

Ứng với $({{a}_{1}},{{a}_{2}},\ldots ,{{a}_{2n}})\in S$, ta có phân loại sau:

  • Tất cả các tổng trong $T$ đều là $0$, đặt số lượng dãy có tính chất này là ${{a}_{n}}$.
  • Trong $T$ có chứa số $2$, đặt số lượng dãy có tính chất này là ${{b}_{n}}$.
  • Trong $T$ có chứa số $-2$, đặt số lượng dãy có tính chất này là ${{c}_{n}}$.

Từ đó, ta dễ dàng chứng minh được hệ thức truy hồi

$ {{a}_{n+1}}=2{{a}_{n}} $

${{b}_{n+1}}={{a}_{n}}+2{{b}_{n}}+{{c}_{n}} $

$ {{c}_{n+1}}={{a}_{n}}+{{b}_{n}}+2{{c}_{n}} $

Chú ý rằng ${{a}_{n}}={{2}^{n}}$ và ${{a}_{n}}+{{b}_{n}}+{{c}_{n}}={{s}_{n}}$. Cộng hai công thức cuối lại, ta có

$${{b}_{n+1}}+{{c}_{n+1}}=2{{a}_{n}}+3({{b}_{n}}+{{c}_{n}})\Leftrightarrow {{s}_{n+1}}-{{2}^{n+1}}={{2}^{n+1}}+3({{s}_{n}}-{{2}^{n}})$$ hay

$${{s}_{n+1}}=3{{s}_{n}}+{{2}^{n}}\Leftrightarrow {{s}_{n+1}}+{{2}^{n+1}}=3({{s}_{n}}+{{2}^{n}}).$$

Với $n=1$, ta có $4$ dãy là $(1,1),(-1,-1),(-1,1),(1,-1)$ nên ${{s}_{1}}=4.$

Từ đẳng thức trên, ta có ${{s}_{n}}+{{2}^{n}}={{3}^{n-1}}({{s}_{1}}+{{2}^{1}})=2\cdot {{3}^{n}}$ nên ${{s}_{n}}=2\cdot {{3}^{n}}-{{2}^{n}}$.

Nhận xét: Bài toán thoạt nhìn có vẻ quen thuộc nhưng thật không đơn giản. Điều kiện đề cho là giá trị tuyệt đối của tất cả các tổng con từ vị trí lẻ đến vị trí chẵn bất kỳ đều không vượt quá $2$ buộc ta phải có đánh giá thích hợp mới có thể truy hồi được.

2. Bài tập áp dụng

Bài 1 (TP.HCM 2018): Hỏi có bao nhiêu hoán vị $(a_1,\ a_2,\ \ldots ,\ a_{164})$ của $164$ số nguyên dương đầu tiên sao cho $a_i \ne i$ và $ a_i \equiv i\text{ }(\bmod 41)$ với mọi $i=1,2,\ldots ,164?$

Bài 2: (Bài toán phát kẹo) Cô giáo có $10$ loại kẹo (mỗi loại có nhiều viên) và cần phát cho $30$ học sinh của lớp (một em nhận không quá $1$ viên/loại), giả sử rằng các em này có học lực đôi một khác nhau. Hỏi cô giáo có bao nhiêu cách phát kẹo, biết rằng nếu học sinh $A$ giỏi hơn $B$ thì $B$ có kẹo gì là $A$ có kẹo đó (tính cả trường hợp không em nào nhận được kẹo)?

Bài 3: (Bài toán con nhện) Một con nhện có $8$ cái chân, $8$ cặp vớ – giày khác nhau (vớ chỉ dùng chung với chiếc giày tương ứng). Con nhện có bao nhiêu thứ tự mang vớ và giày để sao cho trên cùng một chân, giày phải được mang vào sau vớ?

Đa thức bất khả quy

ĐA THỨC BẤT KHẢ QUY

(Thầy Vương Trung Dũng  giáo viên trường PTNK TP Hồ Chí Minh)

1. Giới thiệu sơ lược 

Đa thức bất khả qui là một vấn đề kinh điển trong đa thức nói riêng và trong toán học nói chung. Các bài toán về đa thức bất khả qui cũng thường xuyên xuất hiện trong các kì thi Olympic về toán. Người ta quan tâm nhiều nhất về tính bất khả qui của một đa thức trên vành $\mathbb{Z}[x]$ và $\mathbb{Q}[x]$. Có nhiều cách để kiểm tra tính bất khả qui của một đa thức loại này chẳng hạn như dùng trực tiếp định nghĩa hoặc dùng các tiêu chuẩn như tiêu chuẩn Eisenstein, tiêu chuẩn Perron, tiêu chuẩn Cohn, tiêu chuẩn Dumas… tuy nhiên bài viết này chỉ đề cập đến hai phương pháp thường được sử dụng nhất là sử dụng trực tiếp định nghĩa và tiêu chuẩn Eisenstein và các dạng mở rộng của nó cùng với đó là một kĩ thuật tối quan trọng là rút gọn theo một modulo nguyên tố $p$. Các tiêu chuẩn khác hi vọng sẽ có dịp trình bày trong một bài viết khác.

Trong tài liệu này ta qui ước $\mathbb{Z}_p=\mathbb{Z}/p\mathbb{Z}$ và $\mathbb{K}$ là một trong các tập $\mathbb{Z},\ \mathbb{Q},\ \mathbb{R}, \ \mathbb{Z}_p$. Khi đó, $ \mathbb{K}[x]$ (tương ứng $ \mathbb{K}[x,y]$) là các vành đa thức một biến (tương ứng 2 biến) có hệ số trong $ \mathbb{K}$.

Định nghĩa 1.1:  Đa thức $P(x)$ trong vành $\mathbb{K}[x]$ được gọi là khả qui trên $\mathbb{K}$ nếu $P(x)=f(x).g(x)$ trong đó $f(x), g(x)$ là các đa thức không khả nghịch trong $\mathbb{K}[x]$. Đa thức $P(x)$ được gọi là bất khả qui nếu $P(x)$ không khả nghịch và không khả qui.

Nói riêng, khi $\mathbb{K}$ là một trường thì một đa thức $P(x) \in \mathbb{K}[x]$ có bậc dương được gọi là khả qui trên $\mathbb{K}$ nếu có thể phân tích được thành tích hai đa thức có bậc dương trong $\mathbb{K}[x]$, ngược lại $P(x)$ được gọi là bất khả qui trên $\mathbb{K}$.

Định lí Gauss 1.1: Các vành đa thức

  • $\mathbb{R}[x], \ \mathbb{C}[x],\ \mathbb{Q}[x], \ \mathbb{Z}_p[x]$
  •  $\mathbb{Z}[x], \ \mathbb{Z}[x,y], \ \mathbb{Q}[x,y]…$

là có sự phân tích duy nhất thành các nhân tử bất khả qui và sự phân tích này là duy nhất. Nói riêng các khái niệm về đa thức bất khả qui, ước chung lớn nhất, bội chung nhỏ nhất vẫn còn đúng trên các vành này.

Lưu ý: Trong trường hợp $1$ ở trên là các đa thức có hệ số trên trường nên trên đó thuật toán Euclid hay định lí Bezout vẫn còn đúng nhưng trường hợp $2$ thì không.

2. Tính bất khả qui trên $\mathbb{C}[x]$ và $\mathbb{R}[x]$

Định lí 2.1: Mọi đa thức có bậc lớn hơn 1 đều khả qui trên $\mathbb{C}[x]$.

Chứng minh

Giả sử $degP>1$. Ta thừa nhận định lí cơ bản của đại số “Mọi đa thức $P(x) \in \mathbb{C}[x]$ có bậc lớn hơn 1 đều có ít nhất một nghiệm trên $\mathbb{C}$”. Khi đó $P(x)$ có nghiệm $x_0 \in \mathbb{C}$ nên theo Định lí Bezout $$P(x)=(x-x_0)Q(x),$$

trong đó $deg\ge 1$ nên $P(x)$ khả qui trên $\mathbb{C}[x].$

Định lí 2.2: Mọi đa thức có hệ số thực bậc lớn hơn 2 đều khả qui trên $\mathbb{R}[x]$. Nói riêng một đa thức là bất khả qui trên $\mathbb{R}[x]$ khi và chỉ khi nó là đa thức bậc nhất hoặc bậc 2 vô nghiệm.

Chứng minh

Giả sử $P \in \mathbb{R}[x]$ và $deg P >2$.

  • Nếu $\deg P$ lẻ thì $P$ có ít nhất một nghiệm thực nên nó khả qui.
  • Nếu $\deg P$ chẵn thì $P$ có một nghiệm phức $\alpha$, khi đó $\overline{\alpha}$ cũng là nghiệm của $P$ và do đó $P(x)=(x-\alpha)(x-\overline{\alpha})Q(x)$ là khả qui.

3. Tính bất khả qui trên $\mathbb{Z}[x]$ và $ \mathbb{Q}[x]$

Qua Định lí 2.1 và Định lí 2.2 ta thấy nếu $\mathbb{K}=\mathbb{C},\ \mathbb{R}$ thì tính bất khả quy là đơn giản nên ta quan tâm đến trường hợp $\mathbb{K}=\mathbb{Z}, \ \mathbb{Q}.$ Thật may mắn là bổ đề Gauss mà ta trình bày sau đây sẽ cho ta một sự tương ứng về tính bất khả qui của một đa thức hệ số nguyên trên $\mathbb{Z}[x]$ và $\mathbb{Q}[x]$.

Định nghĩa 3.1: Cho đa thức $P(x)=a_nx^n+a_{n-1}x^{n-1}+…+a_1x+a_0 \in \mathbb{Z}[x]$, đa thức $P$ được gọi là nguyên bản nếu $gcd(a_n,…,a_0)=1$

Mệnh đề 3.1: Tích của hai đa thức nguyên bản là một đa thức nguyên bản.

Mệnh đề 3.2: Mọi đa thức $P \in \mathbb{Q}[x]$ đều viết được dưới dạng $P=cP_0(x)$, trong đó $P_0$ là một đa thức nguyên bản và $c_0 \in \mathbb{Q}.$

Định lí 3.1 (Bổ đề Gauss):  Một đa thức hệ số nguyên, có bậc dương bất khả qui trong $\mathbb{Q}[x]$ khi và chỉ khi nó bất khả qui trong $\mathbb{Z}[x]$.

Chứng minh

Hiển nhiên nếu $P(x)$ bất khả qui trên $\mathbb{Q}[x]$ sẽ bất khả qui trên $\mathbb{Z}[x]$.

Ngược lại giả sử $P(x)$ bất khả qui trên $\mathbb{Z}[x]$ mà

$P(x)=P_1(x)P_2(x)$, với $P_1, P_2 \in \mathbb{Q}[x]$ và $1\le deg \ P_1, degP_2 \le deg\ P$.

Khi đó ta viết lại $P_1=\dfrac{a_1}{b_1}Q_1(x), P_2=\dfrac{a_2}{b_2}Q_2(x)$, với $(a_i,b_i)=1$ và $Q_i$ nguyên bản, $i \in \{1,2\}$.

Suy ra $P(x)=\dfrac{a_1a_2}{b_1b_2}Q_1(x)Q_2(x)=\dfrac{p}{q}Q_1(x)Q_2(x),$ trong đó $p=a_1b_1, q=a_2b_2$ và $ (p,q)=1.$

Do $P\in \mathbb{Z}[x]$ nên các hệ số của $Q_1(x)Q_2(x)$ phải chia hết cho $q$ điều này trái với tính nguyên bản của $Q_1(x)Q_2(x)$.

Từ đó ta có điều phải chứng minh.

Định lí 3.2: Cho $P(x)=a_nx^n+a_{n-1}x^{n-1}+…+a_1x+a_0 \in \mathbb{Z}[x]$. Giả sử $P$ có nghiệm hữu tỉ $x=\dfrac{p}{q}$ với $(p,q)=1$. Khi đó, $p$ là ước của $a_0$ còn $q$ là ước của $a_n.$ Nói riêng, mọi nghiệm hữu tỉ của một đa thức monic (đơn khởi, hệ số của bậc cao nhất bằng $\pm1$) với hệ số nguyên đều là nghiệm nguyên.

Chứng minh

Giả sử $P(x)$ có nghiệm hữu tỉ $\dfrac{p}{q},$ với $ (p,q)=1$. Khi đó $$a_n(\dfrac{p}{q})^n+…+a_1. \dfrac{p}{q}+a_0=0,$$

qui đồng mẫu số ta được $$a_np^n+…+a_0q^n=0.$$

Vì vế phải chia hết cho $p$ nên vế trái chia hết cho $p$, từ đó suy ra $a_0q^n$ chia hết cho $p$, lại có $(q^n,p)=1$ nên $a_0$ chia hết cho $p$. Lập luận tương tự ta được $a_n$ chia hết cho $q$.

Đinh lí 3.3: Cho $P \in \mathbb{Q}[x]$ có bậc 2 hoặc 3. Khi đó, $P(x)$ là bất khả qui khi và khi khi $P(x)$ không có nghiệm hữu tỉ.

Chứng minh

Hiển nhiên nếu $P$ có nghiệm hữu tỉ thì nó khả qui.

Đảo lại, nếu $P$ khả qui thì $P$ phân tích được thành tích của hai đa thức hữu tỉ.

Điều kiện bậc của $P$ bằng 2 hoặc 3 chứng tỏ một trong hai nhân tử trên phải có bậc 1.

Từ đó suy ra $f$ có nghiệm hữu tỉ.

Lưu ý: Định lí trên vẫn còn đúng nếu ta thay $\mathbb{Q}$ bởi một trường $\mathbb{K}$ bất kì. Tức là, đa thức $f \in \mathbb{K}[x]$ với bậc bằng 2 hoặc 3 là bất khả qui nếu và chỉ nếu nó không có nghiệm trong $\mathbb{K}.$

Dưới đây là một số ví dụ

Ví dụ 3.1 (Định lí Schur): Cho các số nguyên phân biệt $a_1, a_2,…,a_n$. Khi đó đa thức

a) $f(x)=(x-a_1)(x-a_2)…(x-a_n)-1$ là bất khả qui trên $\mathbb{Q}[x]$.

b) $f(x)=(x-a_1)(x-a_2)…(x-a_n)+1$ là bất khả qui trên $\mathbb{Q}[x]$ ngoại trừ các trường hợp

  • $(x-a)(x-a-2)+1=(x-a-1)^2$,
  • $(x-a)(x-a-1)(x-a-2)(x-a-3)+1=[(x-a-1)(x-a-2)+1]^2.$
Giải

a) Giả sử $(x-a_1)(x-a_2)…(x-a_n)-1=g(x)h(x)$, với $1 \le deg \ f, \deg \ g \le n-1$ và $g, h \in \mathbb{Z}[x].$

Ta có $g(a_i)h(a_i)=-1$ với mọi $i$, từ đó do $g(a_i), h(a_i)$ là các số nguyên nên $ (g+h)(a_i)=0,$ với mọi $i=1,2,…,n.$

Nhưng vì $deg \ (g+h) \le n-1$ triệt tiêu tại $n$ giá trị phân biệt nên $g \equiv – h$.

Từ đó $$(x-a_1)(x-a_2)…(x-a_n)-1=-(g(x))^2.$$

Đẳng thức trên là vô lí vì hệ số cao nhất ở hai vế trái dấu.

b) Lập luận hoàn toàn như trên, giả sử $f(x)$ là khả qui, bằng một phép đổi biến đơn giản ta hoàn toàn có thể viết lại $f$ dưới dạng $$f(x)=x(x-a_1)(x-a_2)…(x-a_{n-1})+1=g(x).h(x),$$ trong đó $0<a_1<a_2<…<a_{n-1}$ và $1 \le g, h \in \mathbb{Z}, \deg(g), \deg(h) \le n-1$.

Từ đẳng thức $g(a_i)h(a_i)=1$ ta suy ra $g(a_i)=h(a_i)= \pm 1$ với mọi $i$ và đẳng thức này xảy ra tại $n$ giá trị phân biệt. Điều đó dẫn dến $g(x)=h(x)$ và ta có $f(x)=g^2(x)$.

Nói cách khác, $\deg(f)=n$ là một số chẵn. Khi đó $f(\dfrac{1}{2})=\dfrac{1}{2}(\dfrac{1}{2}-a_1)…(\dfrac{1}{2}-a_{n-1})+1=1-\dfrac{1}{2^n}(2a_1-1) \ldots (2a_{n-1}-1) \le 1-\dfrac{1}{2^n}1.3 \ldots (2n-3) <0$ với mọi $n \ge 6$ (vô lí). Như vậy $n=2$ hoặc $n=4$.

  • Nếu $n=2$ thì $f(\dfrac{1}{2})=1-\dfrac{1}{4}(2a_1-1) \Rightarrow a_1 \le \dfrac{5}{2}$ và do đó $a_1=1, 2$. Giá trị $a_1$ cho ta $f(x)=x(x-1)+1$ là bất khả qui và $a_1=2$ cho ta $f(x)=x(x-2)+1=(x-1)^2$ là khả qui.
  • Nếu $n=4$ thì $f(\dfrac{1}{2})=1-\dfrac{1}{16}(2a_1-1)(2a_2-1)(2a_3-1) \Rightarrow 0 \le \dfrac{1}{16}(2-1)(3-1)(2a_3-1) \Rightarrow a_3 \le \dfrac{19}{6}$. Xét trường hợp $a_1=1, a_2=2, a_3=3$ ta được $$f(x)=x(x-1)(x-2)(x-3)+1=(x^2-3x+1)^2$$ là khả qui.

Bài toán được chứng minh xong.

Ví dụ 3.2: Cho $a_1, a_2,…,a_n$ là các số nguyên dương phân biệt. Chứng minh rằng đa thức $$P(x)=(x-a_1)^2(x-a_2)^2…(x-a_n)^2+1$$ là bất khả qui trên $\mathbb{Z}.$

Giải

Giả sử $P(x)$ là khả qui, tức tồn tại hai đa thức $G(x), H(x) \in \mathbb{Z}[x]$ có bậc không bé hơn 1 sao cho $P(x)=G(x).H(x)$.

Ta có $P(a_i)=G(a_i).H(a_i)$ với $i=1,2,…,n$ nên $G(a_i)=H(a_i)= \pm 1.$ Ta xét các trường hợp

  • Nếu $deg \ G= \ deg \ H$ thì $deg(G-H) \le n-1 \Rightarrow G \equiv H.$ Từ đó $P(x)=(G(x))^2 \Leftrightarrow 1= \Big(G(x)-(x-a_1)…(x-a_n) \Big)\Big(G(x)+(x-a_1)…(x-a_n) \Big)$ , vô lí vì bậc vế phải luôn không nhỏ thua 1.
  • Nếu $degH<deg G$ thì $degH<n$ mà $H(a_i)= \pm1, i=1,2,…,n$ dẫn đến $H$ là đa thức hằng, vô lí.

Vậy $P(x)$ bất khả qui.

4. Rút gọn modulo $p$ nguyên tố

Kĩ thuật rút gọn modulo $p$ nguyên tố là một kĩ thuật tối quan trọng trong việc chứng minh một đa thức là bất khả qui trên $\mathbb{Z}$. Nó đưa các hệ số từ một trường vô hạn các phần tử về một trường hữu hạn các phần từ, từ đó các tính toán của ta có thể được đơn giản hơn.

Định nghĩa 4.1: Cho $P(x)= \sum \limits_{i=0}^n a_ix_i \in \mathbb{Z}[x], a_n \ne 0$ và $p$ là số nguyên tố. Giả sử $p$ không phải là ước của $gcd(a_1,a_2,…,a_n)$. Ta kí hiệu $\overline{P}$ là đa thức nhận được từ $P$ bằng cách rút gọn các hệ số theo modulo $p$ (lúc này $P(x) \in \mathbb{Z}_p[x]$). Khi đó ta gọi $\overline{P}$ là \textit{đa thức rút gọn theo modulo} $p$ của $P.$

Từ định nghĩa trên ta có sự kiện sau là hiển nhiên $$\overline{P+Q}=\overline{P}+\overline{Q}$$

$$\overline{PQ}=\overline{P}. \ \overline{Q}$$

Định nghĩa 4.2: Nếu đa thức rút gọn modulo $p$ của P bất khả qui thì ta nói đa thức $P$ bất khả qui $mod \ p.$

Định lí 4.1: Với mỗi $P(x) \in \mathbb{Z}[x]$, tồn tại các đa thức $P_1(x), P_2(x), …, P_k(x) \in \mathbb{Z}_p[x]$ sao cho $$\overline{P}(x)=P_1(x).P_2(x)…P_k(x),$$

sự phân tích này là duy nhất theo modulo $p$.

Định lí 4.2: Cho $P(x)= \sum \limits_{i=0}^n a_ix_i \in \mathbb{Z}[x], a_n \ne 0$ và $p$ không là ước của $a_n$. Khi đó, nếu $P(x)$ là bất khả qui $mod \ p$ thì $P(x)$ là bất khả qui. Điều ngược lại của định lí nói chung không đúng.

Chứng minh

Giả sử $P(x)=a_nx^n+a_{n-1}x^{n-1}+…+a_1x+a_0 \in \mathbb{Z}[x]$ và $p$ không là ước của $a_n$.

Giả sử $P(x)=f(x).g(x)$ với $f, g \in \mathbb{Z}[x]$ với $deg \ f, g \ge 1$.

Khi đó $\overline{P}=\overline{f}. \overline{g}$. Do $p$ không là ước của $a_n$ nên bậc của các đa thức $P, f, g$ không thay đổi sau khi rút gọn theo modulo $p$.

Điều này chứng tỏ $\overline{P}$ khả qui theo modulo $p$, vô lí. Ta có điều phải chứng minh.

Ngược lại dễ thấy đa thức $P(x)=x^4+1$ bất khả qui trên $\mathbb{Z}[x]$ nhưng khả qui modulo $p$ với mọi số nguyên tố $p$.

Ví dụ 4.1: Chứng minh đa thức $P(x)=x^5+4x^4+2x^3+5x^2-7$ là bất khả qui.

Giải
  •  Rút gọn theo modulo 2 ta được $\overline{P}(x)=x^5+x^2+1.$
  •  Giả sử $\overline{P}(x)=f(x). g(x)$, với $f, g \in \mathbb{Z}_2$.
  •  Nếu $deg \ f=1$ hoặc $deg \ g=1$ dễ thấy là vô lí vì $\overline{P}$ không có nghiệm trong $\mathbb{Z}_2$.
  •  Suy ra $\overline{P}(x)=(x^2+ax+b)(x^3+cx^2+dx+e)$, với $a,b,c,d,e \in \mathbb{Z}_2$. Đồng nhất hệ số hai vế ta được điều vô lí. Từ đó suy ra điều phải chứng minh.

Ta có thể liệt kê ra các đa thức bất khả qui modulo 2 trong một số trường hợp bậc nhỏ như sau

  • Trường hợp $n=1$ gồm các đa thức: $x, x+1$.
  •  Trường hợp $n=2$ chỉ gồm một đa thức: $1+x+x^2$.
  •  Trường hợp $n=3$ gồm các đa thức: $1+x+x^3, 1+x^2+x^3$.
  •  Trường hợp $n=4$ gồm các đa thức: $1+x+x^4, 1+x+x^2+x^3+x^4$.
  •  Trường hợp $n=5$ gồm các đa thức:

$1+x+x^2+x^4+x^5$,

$1+x+x^3+x^4+x^5$,

$1+x^2+x^3+x^4+x^5$,

$1+x+x^2+x^3+x^5$,

$1+x^3+x^5, 1+x^2+x^5.$

5. Tiêu chuẩn Eisenstein và một số dạng mở rộng

Khi kiểm tra đa tính bất khả qui của một đa thức trên $\mathbb{Z}[x]$ tiêu chuẩn Eisenstein cung cấp cho ta một công cụ hiệu quả.

Định lí 5.1 (Tiêu chuẩn Eisenstein): Cho đa thức $P(x)= \sum \limits_{i=0}^na_ix^i \in \mathbb{Z}[x], a_n \ne 0$. Khi đó nếu tồn tại số nguyên tố $p$ thỏa đồng thời các điều kiện

  • $p$ không là ước của $a_n$;
  • $p$ là ước của $a_i$ với mọi $i\in \{1,2,…,n-1\}$;
  • $p^2$ không là ước của $a_0.$

Khi đó $P(x)$ là đa thức bất khả qui trên $\mathbb{Q}[x].$

Chứng minh

Có rất nhiều cách chứng minh cho tiêu chuẩn Eisenstein, ở đây ta sẽ trình bày chứng minh bằng cách rút gọn theo một modulo $p$ nguyên tố bất kì.

Giả sử $f$ khả qui, tức $f(x)=g(x).h(x)$, với $f, g \in \mathbb{Z}[x]$ và $deg \ f, g \ge 1.$ Rút gọn theo modulo $p$ nguyên tố ta được đẳng thức trong $\mathbb{Z}_p[x]$ dưới dạng $$\overline{f}=\overline{g}. \overline{h}.$$

Từ điều kiện $p$ là ước của $a_0, …, a_{n-1}$ nhưng không là ước của $a_n$ ta suy ra $\overline{f}=\overline{a_n}x^n.$

Từ đó suy ra $\overline{g}= \overline{b_k}x^k, \overline{h}=\overline{b_m}x^m$. Điều này có nghĩa là $\overline{b_0} \equiv…\equiv \overline{b_{k-1}} \equiv \overline{c_0} \equiv… \equiv \overline{c_{m-1}} \equiv 0 \ mod(p)$. Nhưng khi đó $a_0=b_0c_0 \equiv0 \ (mod \ p^2)$ (vô lí). Từ đó ta có điều phải chứng minh. $\square$

Ví dụ 5.1: Chứng minh đa thức $P(x)=x^4-x^2+2x+1$ bất khả qui trên $\mathbb{Z}$.

Giải

Đặt $Q(x)=P(x+1)=x^4+3x^3+3x^2+3x+3$. Khi đó theo tiêu chuẩn Eisenstein với $p=3$ ta có điều phải chứng minh.

Định lí 5.2 (Dạng mở rộng thứ nhất của tiêu chuẩn Eisenstein):

Cho $f(x)=a_nx^n+a_{n-1}x^{n-1}+…+a_1x+a_0 \in \mathbb{Z}[x]$. Giả sử tồn tại số nguyên tố $p$ thỏa mãn với một số tự nhiên $k \le n$ nào đó mà

  • p không là ước của $a_k$;
  • $p$ là ước của $a_0, …, a_{k-1}$;
  • $p^2$ không là ước của $a_0$.

Thế thì $f(x)$ có một nhân tử bất khả qui bậc $ \ge k$ ( và do đó nếu không bất khả qui sẽ có một nhân tử bậc $\le n-k$)

Chứng minh: Bạn đọc có thể tự chứng minh như trong trường hợp nguyên bản của định lí.

Ví dụ 5.2: Chứng minh đa thức $f(x)=x^{101}+101x^{100}+102$ là bất khả qui.

Giải

Áp dụng tiêu chuẩn Eisenstein mở rộng cho $p=2, k=100$ ta thấy, nếu $f$ là khả qui thì nó phải có một nhân tử bậc 1 và do đó $f$ phải có nghiệm hữu tỉ. Nói riêng vì hệ số bậc cao nhất bằng 1 nên nghiệm hữu tỉ này phải là nghiệm nguyên. Dễ thấy điều này là không xảy ra. bài toán được chứng minh xong.

 

Định lí 5.3 (Dạng mở rộng thứ hai của tiêu chuẩn Eisenstein):

Cho $f(x)=a_nx^n+a_{n-1}x^{n-1}+…+a_1x+a_0 \in \mathbb{Z}[x]$. Giả sử tồn tại số nguyên tố $p$ thỏa mãn với một số tự nhiên $k \le n$ nào đó mà

  • p không là ước của $a_n$;
  • $p$ là ước của $a_0, …, a_{n-1}$;
  • $p^2$ không là ước của $a_k$.

Thế thì, hoặc $f(x)$ là bất khả qui, hoặc $f$ có một nhân tử bất khả qui bậc $ \le k.$

Tương tự như trên, chứng minh được dành cho bạn đọc.

6. Các bài toán áp dụng

Bài tập 6.1 (IMO 1993): Cho số tự nhiên $n>1$. Chứng minh đa thức $f(x)=x^n+5x^{n-1}+3$ là bất khả qui trên $\mathbb{Z}[x]$.

Giải

Áp dụng dạng mở rộng thứ nhất của tiêu chuẩn Eisenstein với $p=3, k=n-1$ ta có điều phải chứng minh.

Bài tập 6.2 (China TST 1994): Cho số tự nhiên $n \ge 3$ và hai số nguyên tố $p, q$ phân biệt. Tìm tất cả các số nguyên $a$ sao cho đa thức $P(x)=x^n+ax^{n-1}+pq$ bất khả qui trên $\mathbb{Z}.$

Giải

Nếu $p|a$ hoặc $q|a$ thì theo tiêu chuẩn Eisenstein $P(x)$ là bất khả qui. Xét trường hợp $p,q$ đều không là ước của $a$. Giả sử $P$ khả qui, áp dụng dạng mở rộng thứ nhất của tiêu chuẩn Eisenstein ta suy ra $P(x)$ phải có nhân tử bậc 1 và do đó $P$ có một nghiệm nguyên $x_0$.

Từ đó suy ra $pq=-x_0^{n-1}(x_0+a)$. Vì $n \ge 3$ nên $pq \ \vdots \ x_0^2$ nhưng vì $p \ne q$ nên $x_0=\pm 1.$

Vì $1+a+pq=0$ và $(-1)^n+a(-1)^{n-1}+pq=0$ nên $a=-1-pq$ hoặc $a=1+(-1)^npq.$

Bài tập 6.3 (Rumani TST 1998): Chứng minh rằng đa thức $P(x)=(x^2+x)^{2^n}+1$ là bất khả qui với mọi số tự nhiên $n$.

Giải

Bằng qui nạp ta chỉ ra rút gọn modulo 2 thì đa thức đã cho trở thành $(x^2+x+1)^{2^n}$. Chú ý rằng đa thức $x^2+x+1$ là bất khả qui modulo 2.

Giả sử $P(x)$ khả qui, tức tồn tại hai đa thức $f,h$ đơn khởi với $f, g \in \mathbb{Z}[x], deg \ f,g \ge 1$ sao cho $P(x)=f(x).g(x)$. Khi đó $\overline{f}=(x^2+x+1)^k, \overline{g}=(x^2+x+1)^{2^n-k}$ và $$f(x)=(x^2+x+1)^k+2f_0(x), g(x)=(x^2+x+1)^{2^n-k}+2g_0(x),$$

với $f_0, g_0 \in \mathbb{Z}[x]$.

Gọi $j$ là một căn bậc 3 khác 1 của đơn vị. Thay $j$ vào đẳng thức $P(x)=f(x).h(x)$ ta được $$P(j)=g(j).h(j) \Leftrightarrow 2=4f_0(j)g_0(j).$$

Từ đó suy ra $f_0(j).g_0(j)=\dfrac{1}{2}$.

Do $f_0(j)g_0(j)$ luôn viết được dưới dạng $aj+b; a, b \in \mathbb{Z}$ và đẳng thức này không thể xảy ra. Ta có điều phải chứng minh.

Một số bài toán tương tự như sau:

Bài 1: Với $n \ge 1$ là số tự nhiên, chứng minh các đa thức sau là bất khả qui trên $\mathbb{Z}$

a) $P(x)=(x^3+x)^{2^n}-3$

b) $P(x)=(x^2+ax)^{2^n}+1$ với $ a \in \mathbb{Z}$

Bài 2: Cho $p$ là một số nguyên tố có dạng $4k+3$. Chứng minh rằng với mọi số nguyên dương $n$ đa thức $P(x)=(x^2+1)^n+p$ bất khả qui trên $\mathbb{Z}[x]$.

Bài 3: Cho $p$ là một số nguyên tố và $a$ là một số nguyên không chia hết cho $p$. Chứng minh đa thức $P(x)=x^p-x+a$ bất khả qui trên $\mathbb{Z}[x].$

Bài tập 6.4 (Japan 99): Chứng minh rằng đa thức $f(x)=(x^2+1^2)(x^2+2^2)…(x^2+n^2)+1$ là bất khả qui trên $\mathbb{Z}$

Giải

Giả sử $n \ge 2$ vì trường hợp $n=1$ là tầm thường và giả sử $f(x)=g(x).h(x)$ với $ f, g \in \mathbb{Z}[x]$ và $1 \le deg \ f, g \le n-1$. Khi đó $$1=f(\pm ki)=g(\pm ki) h(\pm ki).$$

Vì $f, g \in \mathbb{Z}[x]$ nên $g(\pm ki), h(\pm ki)$ có dạng $a + bi$ . Từ đó suy ra $$1=g(ki)h(ki)=1.1=(-1).(-1)=i.(-i)=(-i).i$$

Như vậy trong tất cả $4$ trường hợp ta đều có $g(ki)=\overline{h(ki)}=h(-ki),$ với $k=1,2,…,n$. Như vậy đa thức $P(x)=g(x)-h(-x)$ có $2n$ nghiệm phân biệt nhưng có bậc nhỏ hơn $2n$ nên là đa thức 0 và do đó $g(x)=h(-x)$. Suy ra $\deg(g)=\deg(h)=n $.

Vì $f$ đơn khởi nên $g,h$ cũng đơn khởi. Khi đó $g^2-h^2$ có bậc không quá $2n-1$ nhưng lại có ít nhất $2n$ nghiệm $ki$, với $i \in \{-n, -n-1,…,-1,1,…,n\}$ nên $g^2=h^2$.

Nhưng dễ thấy $g=-h$ không xảy ra do đó $g\equiv h$. Khi đó $$f(x)=g(x)^2 \Rightarrow f(0)=(g(0))^2=(n!)^2+1,$$ vô lí. Bài toán được chứng minh xong.

Ta có bài toán tổng quát hơn là: Cho $p$ là một số nguyên tố. Chứng minh rằng với mỗi số tự nhiên $n$ đa thức $$P(x)=(x^p+1^2)(x^p+2^2)…(x^p+n^2)+1$$ bất khả qui trên $\mathbb{Z}[x].$

Bài tập 6.5: Cho $m, n, a$ là các số nguyên dương và số nguyên tố $p$ thỏa mãn $p<a-1$. Chứng minh rằng đa thức $P(x)=x^m(x-a)^n+p$ bất khả qui trên $\mathbb{Z}$.

Giải

Giả sử $P(x)$ khả qui và $P(x)=G(x).H(x)$, với $G, H \in \mathbb{Z}[x]$. Vì $P(0)=p$ nên $|G(0)|=1$ hoặc $|H(0)|=1$. Không mất tổng quát ta giả sử $G(x)=x^k+a_{k-1}x^{k-1}+…+a_0$ và $|G(0)|=1, m+n-1 \ge k \ge 1.

Gọi $x_1,…,x_k$ là nghiệm của $G(x)$. Ta viết $G(x)$ dưới dạng $G(x)=(x-x_1)…(x-x_k)$, dẫn đến $|x_1…x_k|=1$ và $P(x_i)=0 \Leftrightarrow x_i^m(x_i-a)^n=-p$.

Cho $i=\overline{1,k}$ và nhân các vế của đẳng thức lại ta được $$ |G(x)|^n=p^k \ \text{nên} \ |G(a)|^n=p^k.$$

Mặt khác $P(a)=G(a).H(a)=p$ nên ta suy ra $|G(a)|=p$. Do đó $|G(a)-G(0)|=p \pm 1$ chia hết cho $a$.

Vì thế nên $p-1 \ge a$ hoặc $p+1 \ge a$ (mâu thuẫn với giả thiết $p<a-1$).

Vậy $P(x)$ là bất khả qui.

Bài tập 6.6 (Rumani 1999): Cho số nguyên $a$ và số nguyên dương $n$ và $p$ là một số nguyên tố thoả $p>|a|+1$. Chứng minh rằng đa thức $P(x)=x^n+ax+p$ bất khả qui trên $\mathbb{Z}[x]$.

Giải

Giả sử $P(x)=g(x).h(x)$, với $g, h \in \mathbb{Z}[x]$ và $1 \le deg f, deg g \le n-1$. Vì $P(0)=p=g(0).h(0)$ nên không mất tổng quát giả sử $g(0)=\pm 1, h(0)= \pm p.$

Khi đó $g(x)=\pm x^m+T(x)\pm 1, T \in \mathbb{Z}[x], deg T \le m-1.$

Gọi $z_1, z_2,\ldots,z_m$ là nghiệm của $g(x)$. Theo định lí Viet $1=|g(0)|=|z_1z_2…z_m|$ nên $|z_i| \le 1, i=1,2,…,m.$

Lại có $0=f(z_i)=z_i^n+az_i+p$ nên $$p=-z_i^n+az_i \le |z_i|^n+|a|.|z_i| \le 1+|a|,$$

vô lí. Vậy ta có điều phải chứng minh.

Bài tập 6.7: Cho $p, q$ là hai số nguyên tố lẻ sao cho $q$ không là ước của $p-1$ và gọi $a_1, a_2,…,a_n$ là các số nguyên phân biệt sao cho $q|(a_i-a_j)$ với mọi cặp $(i,j)$. Chứng minh rằng $$P(x)=(x-a_1)(x-a_2)…(x-a_n)-p$$ là bất khả qui trên $\mathbb{Z}[x]$ với mọi $n \ge 2.$

Giải

Giả sử $f$ khả qui trên $\mathbb{Z}[x]$. Khi đó tồn tại $Q(x), R(x) \in \mathbb{Z}[x]$ sao cho $Q(x)R(x)=P(x)$ và $1 \le deg Q(x), \le deg R(x).$ Nói riêng $degQ(x) \le \dfrac{n}{2}$.

Ta có $Q(a_i)=R(a_i)=-p,$ với $1 \le i \le n$ từ đó suy ra $Q(a_i), R(a_i) \in \{-1,1,-p,p \}$ với mọi $1 \le i \le n$. Với mọi hằng số $c$ đa thức $Q(x)-c$ nhận nhiều nhất $degQ(x) \le \dfrac{n}{2}$ nghiệm. Do đó $Q(a_i)$ nhận ít nhất hai giá trị phân biệt (và ít nhất $3$ giá trị phân biệt nếu $degQ(x)< \dfrac{n}{2}$).

Vì $q|(a_i-a_j)$ nên $q|(Q(a_i)-Q(a_j)$. Để ý rằng $q$ là số nguyên tố lẻ nên $Q(a_i)$ không thể nhận hai giá trị $1$ và $-1$ (vì nếu ngược lại thì $q|1-(-1)=2$). Tương tự $Q$ cũng không thể nhận hai giá trị là $p -p$ vì khi đó $R(a_i)$ nhận hai giá trị là $1,-1$.

Từ giả thiết $q$ không là ước của $p-1$ suy ra $Q(a_i)$ không thể nhận hai giá trị $1$ và $p$ hoặc $-1$ và $-p$. Do đó $Q(a_i)$ chỉ có thể nhận được nhiều nhất hai giá trị là $1$ và $-p$ hoặc $-1$ và $p$. Giả sử trường hợp đầu tiên xảy ra.

Vì $Q(a_i)$ chỉ nhận hai giá trị nên $degQ(a_i)=\dfrac{n}{2}$ và $Q$ nhận mỗi giá trị $1$ và $-p$ đúng $\dfrac{n}{2}$ lần. Chia tập $(a_i)_{i=1}^n$ thành hai tập $(b_i)_{i=1}^n$ và $(c_i)_{i=1}^n$ sao cho $Q(b_i)=1$ và $Q(c_i)=-p$. Khi đó $$Q(x)=(x-b_1)(x-b_2)…(x-b_{\frac{n}{2}})+1=(x-c_1)(x-c_2)…(x-c_{\frac{n}{2}})-p.$$

Vì ta cũng có $degR(x)=\dfrac{n}{2}$ nên $R(a_i)=-p$ khi $Q(a_i)=1$. Do đó $$R(x)=(x-b_1)(x-b_2)…(x-b_{\frac{n}{2}})-p=(x-c_1)(x-c_2)…(x-c_{\frac{n}{2}})+1.$$

Nhưng khi đó để ý rằng trong đẳng thức thứ nhất cho ta $Q(x)-R(x)=1+p$ còn đẳng thức thứ 2 cho ta $Q(x)-R(x)=-p-1$ điều đó dẫn đến $p=-1$. Vô lí. Bài toán được chứng minh xong.

Bài tập 6.8: Tìm tất cả các cặp số nguyên dương $(m,n)$ sao cho đa thức $$P(x,y)=(x+y)^2(mxy+n)+1$$ khả qui trên $\mathbb{Z}[x,y]$. Khi đó hãy phân tích $f$ thành các nhân tử bất khả qui.

Giải

Đặt $S=x+y$ ta viết lại $f$ dưới dạng $$f(x,S)=S^2(mx(S-x)+n)+1=-mS^2x^2+mS^3x+(nS^2+1).$$

Ta xem $f$ là một tam thức bậc 2 theo biến $x$. Khi đó $f$ khả qui khi và chỉ khi $f$ phân tích được thành tích của hai đa thức bậc nhất. Khi đó biệt thức $$\Delta =m^2S^6+4mS^2(nS^2+1)=mS^2(mS^4+4nS^2+4)$$ là một bình phương.

Dễ thấy điều này xảy ra khi và chỉ khi $m=n^2$, lúc này $$\Delta= (nS(nS^2+2))^2$$ và $f$ có hai nghiệm là $x=\dfrac{-1}{nS}$ hoặc $x=\dfrac{nS^2+1}{nS}$.

Khi đó $$f(x)=(nSx+1)(-nSx+nS^2+1)=(nx^2+nxy+1)(ny^2+nxy+1).$$

 

7. Bài tập tự luyện

Bài 1: Với $n \ge 2$ là một số nguyên và $r=\sqrt[n]{2}$. Chứng minh rằng không tồn tại các số hữu tỷ $a_0, a_1,…,a_{n-1}$ không đồng thời bằng $0$ sao cho $$ a_0+a_1r+a_2r^2+…+a_{n-1}r^{n-1}=0 $$

Bài 2: Tìm số nguyên dương $n$ nhỏ nhất sao cho đa thức $P(x)=x^{n-4}+4n$ có thể phân tích được thành tích của 4 đa thức hệ số nguyên và không là đa thức hằng.

Bài 3: Cho $P(x), Q(x)$ là hai đa thức đơn khởi, bất khả quy trên trường số hữu tỷ. Giả sử $P, Q$ có hai nghiệm tương ứng là $\alpha, \beta$ sao cho $\alpha +\beta$ là số hữu tỷ. Chứng minh $P^2(x)-Q^2(x)$ có nghiệm hữu tỷ.

Bài 4: Chứng minh đa thức $P(x)=(1+x+x^2+…+x^n)^2-x^n$ khả qui trên $\mathbb{Z}[x].$

Bài 5: Chứng minh rằng đa thức $P(x)=x^n+4$ khả qui trên $\mathbb{Z}$ khi và chỉ khi $n$ là bội của $4.$

Bài 6 (IMO Longlist 1989): Cho $n \ge 4$ và các số nguyên phân biệt $a_1,a_2,…,a_n$. Chứng minh đa thức $$P(x)=(x-a_1)(x-a_2)…(x-a_n)-2$$ bất khả qui trên $\mathbb{Q}[x].$

Bài 7 (VMO 2014): Cho $n$ là số nguyên dương. Chứng minh rằng đa thức $P(x)=(x^2-7x+6)^n+13$ không thể biểu diễn được thành tích của $n+1$ đa thức khác hằng với hệ số nguyên.

Bài 8: Chứng minh rằng đa thức $x^n-x-1$ bất khả qui trên $\mathbb{Q}[x]$, với mọi $n \ge 2.$

Bài 9: Cho $n>m>1$ là hai số nguyên lẻ. Chứng minh đa thức $P(x)=x^n+x^m+x+1$ bất khả qui trên $\mathbb{Z}[x]$.

Bài 10: Cho $p$ là số nguyên tố. Chứng minh rằng đa thức $$P(x)=x^{p-1}+2x^{p-2}+…+(p-1)x+p$$ bất khả qui trên $\mathbb{Z}$.

Bài 11: Cho đa thức $P(x)=a_nx^n+a_{n-1}x^{n-1}+…+a_1x+a_0 \in \mathbb{Z}[x], (a_n \ne 0, n \ge 2)$. Chứng minh rằng tồn tại vô số số nguyên tố $k$ sao cho đa thức $P(x)+k$ bất khả qui.

Bài 12: Tìm tất cả các số nguyên $n$ sao cho đa thức $P(x)=x^5-nx-n-2$ là khả qui trên $\mathbb{Z}[x]$.

Bài 13: Cho $p$ là một số nguyên tố và $n$ là một số nguyên nhỏ hơn 4. Chứng minh rằng nếu $a$ là một số nguyên không chia hết cho $p$ thì đa thức $P(x)=ax^n-px^2+px+p^2$ bất khả qui trên $\mathbb{Z}[x].$

Bài 14: Cho $p$ là số nguyên tố. Chứng minh rằng đa thức $P(x)=x^p+(p-1)!$ bất khả qui trên $\mathbb{Z}[x]$.

Bài 15: Tồn tại hay không đa thức $f \in \mathbb{Q}[x]$ sao cho $f(1) \ne -1$ và $x^nf(x)+1$ là khả qui với mọi $n \in \mathbb{N}$.

Bài 16: Cho $a$ là một số nguyên dương và $p \ge 2 $ là một số nguyên tố thỏa mãn $(a,p)=1$. Chứng minh rằng đa thức $P(x)=x^p-mx+a$ bất khả qui trên $\mathbb{Z}[x]$ với $m \equiv \ 1 \ (mod \ p)$.

Bài 17: Cho $p$ là một số nguyên tố lẻ. Chứng minh đa thức $P(x)= \sum \limits_{i=0}^{p-2}(p-1-i)x^i$ bất khả qui trên $\mathbb{Q}[x].$

Bài 18 (Rumani TST 2003): Cho $P(x) \in \mathbb{Z}[x]$ là một đa thức monic bất khả qui trên $\mathbb{Z}[x]$ sao cho $P(0)$ không là số chính phương. Chứng minh rằng $Q(x)=P(x^2)$ cũng bất khả qui trên $\mathbb{Z}[x].$

Bài 19 (China TST 2006): Cho số nguyên $n \ge 2$. Chứng minh rằng tồn tại đa thức $P(x)=x^n+a_{n-1}x^{n-1}+…+a_1x+a_0$ thỏa mãn

a) $a_0, a_1,…,a_{n-1}$ khác 0.

b) $P(x)$ bất khả qui.

c) Với mọi số nguyên $x$ thì $|P(x)|$ không là số nguyên tố.

Bài 20: Biết $f \in \mathbb{Z}[x]$ là một đa thức bất khả qui có bậc lẻ và lớn hơn 3. Giả sử rằng các nghiệm của $P$ đều có modun lớn hơn 1 và $f(0)$ không có ước chính phương. Chứng minh rằng đa thức $g(x)=f(x^3)$ cũng là đa thức bất khả qui.

Bài 21: Cho $f \in \mathbb{Z}[x]$ là một đa thức monic với bậc lớn hơn 1. Giả sử $f(x^n)$ bất khả qui trên $\mathbb{Z}[x]$ với mọi $n \ge 2$. Hỏi $f$ có bất khả qui trên $\mathbb{Z}[x]$ hay không?

Bài 22: Cho $1 \ne f \in \mathbb{Z}[x]$ sao cho có vô hạn số nguyên $a$ thỏa $f(x^2+ax)$ bất khả qui trên $\mathbb{Q}[x]$. Hỏi $f$ có bất khả qui trên $\mathbb{Q}[x]$ hay không?

Bài 23: Cho $f(x) \ne \pm x$ là một đa thức bất khả qui trên $\mathbb{Z}[x]$. Hỏi $f(x.y)$ có bất khả qui trên $\mathbb{Z}[x,y]$ hay không?

Tài liệu tham khảo

[1] Nguyễn Tiến Quang, NXB Giáo dục, Đại số đại cương

[2] Đoàn Duy Cường, 2015, Bài giảng bồi dưỡng giáo viên chuyên toán năm

[3] Nguyễn Chu Gia Vượng,2015,  Đa thức bất khả qui

[4] Exploration-Creativity 2012,  Irreducible polynomials

[5] Yufei Zhao, Integer polynomial 

[6] Dusan Djukic, Polynomials in one variable 

[7] Gabriel D.Carroll, Polynomials 

[8] Victor V.Prasolov, Polynomials

[9] Titu Andresscu, Gabriel Dospinescu, Problems from the book

[10] U298, Mathematical Reflections

[11] https://artofproblemsolving.com/community/c89 

Phương pháp ánh xạ trong các bài toán tổ hợp

Bài viết dựa vào bài giảng của NCS. Vương Trung Dũng (trường PTNK-ĐHQG) trong lớp chuyên đề 10 toán tại Star Education.

 

Ánh xạ là một khái niệm khó và quan trọng trong toán học, có vai trò trong hầu hết các lĩnh vực toán học. Trong bài giảng này ta xét ứng dụng của ánh xạ trong các bài toán tổ hợp.

Ánh xạ và một số tính chất

Định nghĩa. Cho hai tập hợp $X$ và $Y$ khác rỗng. Một ánh xạ $f$ từ tập $X$ đến tập $Y$ là một quy tắc đặt tương ứng mỗi phần tử $x$ của $X$ với một và chỉ một phần tử $y$ của $Y$, kí hiệu là $y = f(x)$.

Kí hiệu $f: X \longrightarrow Y$.

$f(x) = y$.

Các khái niệm: Cho ánh xạ $f: X \longrightarrow Y$.

  • $y = f(x)$ được gọi là ảnh của $x$.
  • $f(X) = \{f(x)|x \in X\}$ tập ảnh của $f$.
  • $y \in Y$ thì $f^-1(y) = \{x\in X|f(x) = y\}$ được gọi là tạo ảnh của $y$.

Đơn ánh, toàn ánh, song ánh

  1. Ánh xạ $f: X \longrightarrow Y$ được gọi là đơn ánh nếu với $a,b \in X$ mà $a \ne b$ thì $f(a) \ne f(b)$. Nói một cách khác ánh xạ $f$ là một đơn ánh nếu và chỉ nếu với $a, b \in X$ mà $f(a)=f(b)$ thì suy ra $a=b.$
  2. Ánh xạ $f:X \longrightarrow Y$ được gọi là toàn ánh nếu với mỗi phần tử $y \in Y$ đều tồn tại một phần tử $x \in X$ sao cho $f(x)=y$. Như vậy $f$ là toàn ánh nếu và chỉ nếu $f(X)=Y$.
  3. Ánh xạ $f: X \longrightarrow Y$ được gọi là song ánh giữa $X$ và $Y$ nếu và chỉ nếu nó vừa là đơn ánh và vừa là toàn ánh. Như vậy $f$ là song ánh nếu với mỗi $y \in Y$ tồn tại duy nhất một phần tử $x \in X$ sao cho $y=f(x).$

Ánh xạ và tập hợp

Cho $A = { 1, 2,\cdots, n }$. $X$ là tập khác rỗng. Nếu có một song ánh từ tập $X$ đến $A$ thì ta nói $X$ có $n$ phần tử và kí hiệu $|X| = n$.

Nếu không tồn tại song ánh thì ta nói $X$ có vô hạn phần tử.

  • Nếu tồn tại một song ánh từ $X$ vào tập các số tự nhiên, ta nói $X$ có lực lượng đếm được, ngược lại thì ta nó $X$ có lực lượng không đếm được.
  • Các tập số tự nhiên, số nguyên và hữu tỷ là các tập có lực lượng đếm được.

Định lý Cho $A$ và $B$ là hai tập hợp hữu hạn.

  • Nếu có một đơn ánh $f: X \longrightarrow Y$ thì $|X| \le |Y|.$
  • Nếu có một toàn ánh $f: X \longrightarrow Y$ thì $|X| \ge |Y|.$
  • Nếu có một song ánh $f: X \longrightarrow Y$ thì $|X| = |Y|.$

Ánh xạ và các bài toán đếm, đẳng thức tổ hợp.

Ví dụ 1. Cho tập $X_n = {1, 2, \cdots, n}$, gọi $P(X_n)$ là tập các tập con của $X_n$, và $S_n$ là tập các dãy nhị phân có độ dài $n$. Tìm một song ánh từ $P(X_n)$ vào $S_n$, suy ra số tập con của $X_n$.

Gợi ý

Định nghĩa một ánh xạ $f: P(X_n) \longrightarrow S_n$ như sau:
Với mỗi $S \in P(X_n)$ (tức là $S \subset X_n$) ta đặt $$ f(S)=y_1y_2 \dots y_n$$
trong đó
$$y_i=\begin{cases}
1, y_i \in S&\\
0, y_i \notin S.&
\end{cases}
$$
Ví dụ , nếu $X=\{1; 2; 3; 4; 5\}, S_1=\{4\}, S_2=\{2; 3; 5\}$ thì $f(S_1)=00010, f(S_2)=01101, f(\emptyset)=00000, f(X)=11111$ .
Dễ dàng kiểm tra đây là một song ánh từ $P(X)$ vào $Y$.
Do đó theo nguyên lý song ánh ta có $|P(X)|=|Y|=2^n$.

Ví dụ 2. Hãy tính trung bình cộng của tất cả các số N gồm 2014 chữ số thỏa mãn N chia hết cho 9 và các chữ số của N được lập từ $X={1,2,…,8}$

Gợi ý

Gọi M là tập các số thỏa yêu cầu đề bài.

Ta xây dựng một ánh xạ đi từ M đến M như sau: Với mỗi $N=\overline{a_1a_2…a_{2014}} \in M$ dặt $f(N)=\overline{b_1,b_2,…,b_{2014}}$ với $b_i=9-a_i$ với mọi $i=1,2,…,2014$. Vì $N+f(N)=99…9$ (2014 số 9) chia hết cho 9 và N chia hết cho 9 nên suy ra $f(N)$ cũng chia hết cho 9. Do đó $f$ là một ánh xạ đi từ M vào M. Hơn nữa dễ thấy $f$ là một song ánh. Từ đó suy ra $$ 2\sum_{N \in M}N=\sum_{N \in M}(N+f(N))=|M|.99…9 .$$ Vậy trung bình cộng của các số trong M là $99…9:2.$

Ví dụ 3. Cho tập S gồm tất cả các số nguyên dương trong đoạn $[1,2,…,2002]$. Gọi T là tập hợp tất cả các tập con khác rỗng của S. Với mỗi X thuộc T ký hiệu m(X) là trung bình cộng các phần tử thuộc X. Tính $$ m=\frac{\sum_{X \in T}m(X)}{|T|}. $$

Gợi ý

Xây dựng song ánh $f: T \longrightarrow T$ như sau: với mọi $X \in T $ đặt tương ứng $f(X)=\{2003-x: x \in X\}$.\\
Khi đó $m(X)+m(f(X))=2003$. Do đó $$2 \sum m(X)=\sum (m(X)+m( f(X)))=|T|.2003 \Rightarrow m=\dfrac{\sum m(X)}{|T|}=\dfrac{2003}{2}$$

Ví dụ 4.  Cho $X={1,2,…,n}$. Có bao nhiêu tập con $k$ phần tử của X sao cho trong mỗi tập con không chứa 2 số nguyên liên tiếp.

Gợi ý

Gọi A là tập tất cả các tập con $k $ phần tử của X mà trong mỗi tập không chứa 2 số nguyên liên tiếp và B là tập tất cả các tập con của tập $Y=\{1,2,…, n-(k-1) \}$. Ta xây dựng song ánh từ A đến B như sau: Lấy $S=\{s_1,s_2,…,s_k \} \in A$ (không mất tổng quát có thể giả sử $s_1<s_2<…<s_k$) đặt tương ứng với $f(S)=\{s_1, s_2-1, s_3-2,…, s_k-(k-1) \}$. Dễ chứng minh đây là một song ánh. Từ đó có $C^k_{n-k+1}$ tập thoả yêu cầu đề bài.

Bài tập rèn luyện 

Bài 1. Cho $X={ 1,2,..,n}$. Một tập con $S={s_1,s_2,…,s_k }$ của X ($s_1<s_2<…<s_k$) được gọi là \textit{m- tách được} $(m \in \mathbb{N})$ nếu $s_i-s_{i-1} \ge m; i=1,2,…,k$. Có bao nhiêu tập con m- tách được gồm $k$ phần tử của X, trong đó $0 \le k \le n-(m-1)(k-1)$.

Bài 2. Cho $X={1,2,…,n}$, với mỗi tập con khác rỗng $A_i={a_1,a_2,…,a_i }$ (không mất tổng quát giả sử $a_1>a_2>…>a_i$) ta định nghĩa \textit{tổng hỗn tạp} của $A_i$ là số $m(A_i)=a_1-a_2+a_3-… \pm a_i$. Tính $\sum \limits_{A_i \subset X} m(A_i)$.

Bài 3. Cho số nguyên dương $n$ và $d$ là một ước dương của $n$. Gọi S là tập tất cả những bộ $(x_1,x_2,…,x_n)$ nguyên dương thỏa $0 \le x_1 \le x_2 \le… \le x_n \le n$ và $d| x_1+x_2+…+x_n$. Chứng minh rằng có đúng một nửa các phần tử của S có tính chất $x_n=n$.

Bài 4. Gọi $a_n$ là số các xâu nhị phân độ dài $n$ không chứa ba bit 0, 1, 0 liên tiếp. Gọi $b_n$ là số các xâu nhị phân độ dài $n$ chứa bốn bit 0, 0, 1, 1 hoặc 1, 1, 0, 0 liên tiếp. Chứng minh rằng $b_{n+1}=2a_n$ với mọi số nguyên dương $n$.

Bài 5. Cho các số tự nhiên $k, n, m$ thỏa điều kiện $1<k \le n, m>1$. Hỏi có bao nhiêu chỉnh hợp chập $k: (a_1,a_2,…,a_k)$ của $n$ số tự nhiên đầu tiên mà mỗi chỉnh hợp đều thỏa mãn ít nhất một trong hai điều kiện sau:

i) Tồn tại $i, j \in {1,2,…,k}$ sao cho $i < j$ và $a_i>a_j$.

ii) Tồn tại $i \in {1,2,…,k}$ sao cho $a_i-i$ không chia hết cho $m$.

Bài 6. Cho các số nguyên dương $n, k, p$ với $k \ge 2$ và $k(p+1) \le n.$ Cho $n$ điểm phân biệt cùng nằm trên một đường tròn. Tô tất cả $n$ điểm đó bởi hai màu xanh, đỏ (mỗi điểm được tô bởi một màu) sao cho có đúng $k$ điểm được tô bởi màu xanh và trên mỗi cung tròn mà hai đầu mút là hai điểm màu xanh liên tiếp (tính theo chiều quay kim đồng hồ) đều có ít nhất $p$ điểm được tô màu đỏ. Hỏi có tất cả bao nhiêu cách tô khác nhau?

Bài 7. Gọi $a_n$ là số các xâu nhị phân độ dài $n$ không chứa ba bit 0, 1, 0 liên tiếp. Gọi $b_n$ là số các xâu nhị phân độ dài $n$ chứa bốn bit 0, 0, 1, 1 hoặc 1, 1, 0, 0 liên tiếp. Chứng minh rằng $b_{n+1}=2a_n$ với mọi số nguyên dương $n$.

Bài 8. Trong một hội nghị có $n$ nhà toán học. Biết rằng nếu hai nhà toán học nào đó quen nhau thì họ không quen chung thêm một người nào nữa, còn nếu hai nhà toán học này không quen nhau thì họ quen chung với đúng hai nhà toán học khác nữa. Chứng minh rằng $8n-7$ là số chính phương.

Bài 9. Trong một trại hè toán học có 40 học sinh. Biết rằng cứ 19 học sinh bất kỳ thì đều viết thư hỏi bài một học sinh khác (Nếu học sinh A viết thư hỏi bài học sinh B thì không nhất thiết học sinh B phải viết thư hỏi bài học sinh A và dĩ nhiên A cũng không viết thư hỏi chính mình). Chứng minh rằng trong trại hè này tồn tại một tập $T_0$ gồm 20 học sinh sao cho với mỗi $P \in T_0$ thì 19 người còn lại không đồng thời viết thư hỏi bài P.

Bài 10. Gọi M là số số nguyên dương trong hệ thập phân có $2n$ chữ số trong đó có $n$ chữ số 1 và $n$ chữ số 2. Gọi N là số số nguyên dương có $n$ chữ số trong hệ thập phân trong đó chỉ có các chữ số 1, 2, 3, 4 và số chữ số 1 bằng số chữ số 2. Chứng minh $|M|=|N|.$

(Hết phần 1)

Một số bài toán số học ôn thi vào 10 – P1

Bài 1. Tìm tất cả các số nguyên tố $p$ sao cho tổng các ước dương của $p^4$ là một số chính phương.

Lời giải

  • Theo đề ta có phương trình $1+p+p^2+p^3+p^4 = x^2$.
  • Ta có $(2p^2+p)^2< 4x^2 < (2p^2+p+2)$.
  • Do đó $4x^2 = (2p^2+p+1) = 4p^2+4p^3+4p^2+4p+4$
  • $p^2 -2p – 3 = 0 \Leftrightarrow p=3$.

Bài 2.  Cho $m,n$ là các số nguyên dương thỏa $m+m+1$ là một ước nguyên tố của $2(m^2+n^2)-1$. Chứng minh rằng $m.n$ là một số chính phương.

Lời giải

Ta có $2m^2+2n^2 -1 = (m+n)^2+(m-n)^2 -1 = (m+n-1)(m+n+1) + (m-n)^2$ chia hết cho $m+n-1$,

suy ra $(m-n)^2$ chia hết cho $m+n+1$.

Mà $m+n+1$ nguyên tố, suy ra $(|m-n|,m+n+1) = 1$, do đó $m=n$, suy ra $mn = m^2$ là số chính phương.

Bài 3.  Chứng minh rằng nếu tích của hai số nguyên tố cùng nhau là một số chính phương thì mỗi số cũng là số chính phương.

Lời giải

Cho $ab = x^2$, trong đó $(a,b)=1$.\
Đặt $d = (a,x), a=a’d, x=x’d$ ta có $a’b = x’^2d$. \
Do $(a’,x’^2)=1$ nên $b$ chia hết cho $x’^2$. \
Mặt khác do $(a,b) = 1$ nên $(b,d) = 1$, suy ra $x’^2$ chia hết cho $b$.\
Do đó $b=x’^2$, $a’=d$. Từ đó ta có $a=a’^2, b= x’^2$ là các số chính phương.\
\textbf{Nhận xét} Tương tự nếu $(a,b) = 1$ và $ab = x^k$ thì $a, b$ là lũy thừa bậc $k$ của một số nguyên.\
Đây là một bổ đề rất hay sử dụng.

Bài 4. Cho các số nguyên dương $a, b$ thỏa $2{a^2} + a = 3{b^2} + b$.
a) Tìm $a, b$ biết $a$ và $b$ là hai số nguyên tố cùng nhau.
b) Chứng minh $a-b$ và $2a + 2b + 1$ là các số chính phương.

Lời giải

a) $a(2a+1) = b(3b+1)$. Ta có $3b +1$ chia hết cho $a$ và $2a+1$ chia hết cho $b$.
Đặt $2a + 1 = kb$, suy ra $3b+1 = ka$. Suy ra $6ab + 2a+3b+1 = k^2ab$, suy ra $k = 1, 2$.
Nếu $k = 1$ ta có $2a+1 = b, 3b+1 = a$ (Vô nghiệm).
Nếu $k = 2$ ta có $2a+1 = 2b, 3b+1 = 2a$. (Vô nghiệm).
Phương trình vô nghiệm.
b) Ta có $(a-b)(2a+2b+1) = b^2$.
Giả sử $p$ là ước nguyên tố của $a-b, 2a+2b+1$, suy ra $p|b^2 \Rightarrow p|b$, suy ra $p|a$, suy ra $p|1$ (vô lý).\
Do đó $(a-b,2a+2b+1) = 1$.
Từ đó ta có $a-b, 2a+2b+1$ là các số chính phương.

Bài 5. Tìm tất cả số tự nhiên $a$ để tồn tại các số nguyên tố $p, q, r$ thỏa $$a=\dfrac{p+q}{r}+
\dfrac{q+r}{p}+ \dfrac{p+r}{q}$$.

Lời giải

  •  Nếu trong 3 số có đúng 2 số bằng nhau, giả sử $p = q \neq r$. Khi đó ta có $a = 2(\dfrac{p}{r}+\dfrac{r}{p}) + 2$. Suy ra $\dfrac{2(p^2+r^2)}{pr} = a-2$.

Suy ra $pr|2(p^2+r^2)$, mà $(p,r) = 1$, suy ra $p|2$, suy ra $p=2$. Vô lý.

  • Nếu 3 số đều khác nhau. Ta có $apqr = pq(p+q) + qr(q+r) + pr(p+r)$. Suy ra $p|qr(q+r)$, suy ra $p|p+q+r$.
    Tương tự ta có $q|p+q+r, r|p+q+r$. Suy ra $pqr|p+q+r$.
    Ta có $pqr > 4r$, suy ra $3pqr > 4(p+q+r) > 4pqr$. Vô lý.
  • 3 số bằng nhau, thì $a = 6$.

Bài tập

Bài 1. Cho $m,n$ và $d$ là các số nguyên dương. Chứng minh rằng nếu $mn^2 + 1$ và $m^2n+1$ cùng chia hết cho $d$ thì $m^3+1$ và $n^3+1$ cũng chia hết cho $d$.

Bài 2. Cho $n \geq 3$ là số tự nhiên sao cho $3n+1$ là số chính phương. Chứng minh rằng có thể tìm được các số nguyên dương $a,b, c$ sao cho $$x = \sqrt{1+\dfrac{3n+3}{a^2+b^2+c^2}} $$
là một số nguyên.

Bài 3. Tìm tất cả các số nguyên $n$ sao cho $n = q(q^2-q-1) = r(2r+1)$ với $p, r$ là các số nguyên tố.

Bài tập hình học ôn thi vào 10 – P1

Bài 1. Cho đường tròn tâm $O$ đường kính $AB$. Tiếp tuyến tại $A$ là $d$, tiếp tuyến tại $B$ là $d’$. $C$ là một điểm thuộc đường tròn, tiếp tuyến tại $C$ cắt $d$ và $d’$ lần lượt tại $D$ và $E$, $BC$ cắt $d$ tại $F$.
a) Chứng minh $D$ là trung điểm của $AF$.
b) Gọi $I$ là giao điểm của $BD$ và $CE$. $CI$ cắt $AB$ tại $G$. Chứng minh $CG^2 = GA.GB$.
c) Đường thẳng qua $A$ song song $EG$ cắt đường thẳng qua $B$ song song với $DG$ tại $H$. Chứng minh $D, H, E$ thẳng hàng.

Lời giải

a) Theo tích chất hai tiếp tuyến cắt nhau thì: $DA = DC$,

tam giác $DAC$ cân tại $D$ nên $\angle DCA = \angle DAC$, mà $\angle DAC + \angle DCF = \angle DAC + \angle DFC= 90^0$.

Do đó $\angle DCF = \angle DFC$, suy ra $DC = DF$. \Vậy $DF = DA$, hay $D$ là trung điểm của AF.

b) Ta có $AD||BE$ nên $\dfrac{ID}{IB} = \dfrac{AD}{BE}$, mà $AD = CD, BE = CE$, suy ra $\dfrac{ID}{IB} = \dfrac{CD}{CE}$. Từ đó ta có $CI || BE$, suy ra $IC \bot AB$.

Tam giác ACB vuông tại C, có CG là đường cao nên: $CG^2 = GA.GB$.

c) Ta có $\dfrac{GA}{GB} = \dfrac{CD}{CE} = \dfrac{AD}{BE}$, suy ra $\triangle ADG \backsim \triangle BEG$, do đó: $\angle AGD = \angle BGC$.
$GJ$ cắt $AD$ tại $J$. Ta có $\angle AGD =\angle BDE = \angle AGJ$.
Suy ra $GEJ$ cân tại $G$ và $A$ là trung điểm của $DJ$.
Gọi $H’$ là trung điểm của $DE$. Suy ra $AH’ || GE$.
Tương tự thì $H’B || GD$. Do đó $H’ \equiv H$.
Vậy $H, D, E$ thẳng hàng.

Bài 2. Cho tam giác $ABC (AB <AC)$ có 3 góc nhọn nội tiếp đường tròn tâm $O$. Vẽ 2 đường cao $AD$ và $CE$ của tam giác $ABC$ . Tiếp tuyến tại $A$ của $(O)$ cắt $BC$ tại $M$ . Từ $M$ kẻ tiếp tuyến thứ hai đến $(O)$ ($N$ là tiếp điểm ). Vẽ $CK$ vuông góc với $AN$ tại $K$. Chứng minh $DK$ đi qua trung điểm của đoạn thẳng $BE$.

Lời giải 

Gọi $Q$ là trung điểm đoạn $BC$.
Ta có $\angle AKD = \angle ACB = \angle ANB$, suy ra $DK || BN$, suy ra $\angle ATK = \angle ABN$.

Ta có 5 điểm $A, M, N, O, Q$ cùng thuộc đường tròn. Suy ra $\angle AQM = \dfrac{1}{2}\angle AON = \angle ACN$.

Suy ra $\angle ABN = 180^\circ- \angle ACN = 180^\circ – \angle AQM =\angle AQC$.

Suy ra $\angle ATK = \angle AQC$. Suy ra $ATDQ$ nội tiếp. Suy ra $AT \bot TQ$. Suy ra $T$ là trung điểm BE.

Bài 3. Cho đường tròn $(O)$ ngoại tiếp tam giác $ABC (AB < AC)$. Gọi $I$ là tâm đường tròn nội tiếp tam giác $ABC$ và $M$ là trung điểm cạnh $BC$. Gọi $Q$ là điểm đối xứng của $I$ qua $M$, tia $OM$ cắt $(O)$ tại $D$ và $QD$ cắt $(O)$ tại $T$ ($T$ thuộc cung $BD$ không chứa $A$).
a) Chứng minh rằng $DI = DB = DC$.
b) Đường thẳng qua $I$ song song $QD$ cắt $DO$ tại $K$. Chứng minh $DK.DO = DB^2$.
c) Chứng minh $\angle ACT = \angle DOI$.

Lời giải

b) Vẽ đường kính $DE$. Ta có $DB^2 = DM\cdot  DE $

$IKQD$ là hình bình hành, suy ra $DK = 2DM$.

Mặt khác $DO = \dfrac{1}{2}DE$

Nên $BD^2 = DK\cdot DO$

c)Vì $DB = DI$ nên ta có $DI^2 = DK\cdot DO$, suy ra $\triangle DIK \backsim \triangle DOI$.

Suy ra $\angle DOI = \angle DIK$ ,

mà $\angle DIK = \angle ADT = \angle ACT$.

Bài tập luyện tập

Bài 1. Cho đường tròn (O) và điểm A nằm ngoài đường tròn. Từ A vẽ đến (O) các tiếp tuyến AB và AC với B, C là các tiếp điểm. Trên tia đối của BA lấy điểm D, đường tròn ngoại tiếp ACD cắt (O) tai điểm thứ hai là E. DE cắt (O) tại F khác E. Gọi I là hình chiếu của B trên CD, H là giao điểm của OB và CD.
a) Chứng minh $CF||AC$.
b) Chứng minh tứ giác $IHEF$ nội tiếp.
c) Chứng minh $\angle IED = 2\angle ADC$.

Bài 2. Cho hình vuông ABCD cạnh a. E, F là các điểm thay đổi trên các cạnh CD và BC sao cho $\angle EAF = 45^0$. Gọi G, H lần lượt là giao điểm của AE, AF với BD.
a) Chứng minh rằng 5 điểm C,E, G, H, F cùng thuộc một đường tròn.
b) Chứng minh EF tiếp xúc với một đường tròn cố định.
c) Chứng minh $GH^2 = DG^2 + BH^2$.
d) Chứng minh chu vi tam giác CEF không đổi. Tìm giá trị lớn nhất diện tích của tam giác CEF.

Bài 3. Cho tam giác ABC nhọn nội tiếp đường tròn tâm O bán kính R. Gọi D là hình chiếu của A trên BC và E là điểm đối xứng của A qua O. Gọi F là điểm chính giữa cung BC không chứa A.
a) Chứng minh rằng AF là phân giác góc $\angle DAE$.
b) Chứng minh $AD.AE = AB.AC$ và $S_{ABC} = \dfrac{AB.AC.BC}{4R}$.
c) Vẽ đường kính FG, đường tròn ngoại tiếp tam giác OAG cắt AB và AC tại M, N. Chứng minh BM = CN.

Đề thi và đáp án tuyển sinh vào 10 TPHCM 2020

Đề thi vào lớp 10 TPHCM năm 2020

Bài 1. Cho parabol $ (P): y=\dfrac{1}{4}x^2$ và đường thẳng $ (d): y=-\dfrac{1}{2}x+2 $

a) Vẽ $ (P) $ và $ (d) $ trên cùng hệ trục tọa độ.

b) Tìm tọa độ giao điểm của $ (P) $ và $ (d) $ bằng phép tính.

Giải

a) Bảng giá trị của $(d)$:

Bảng giá trị của $(P)$:

Đồ thị:

b) Phương trình hoành độ giao điểm của $(P)$ và $(d)$:

$\dfrac{1}{4}x^2 = -\dfrac{1}{2}x +2 \Leftrightarrow \dfrac{1}{4}x^2 +\dfrac{1}{2}x-2=0\Leftrightarrow \left[ \begin{array}{l} x=2\Rightarrow y=1\\ x=-4\Rightarrow y=4 \end{array}\right. $

Vậy tọa độ giao điểm $\left( 2;\, 1\right) $, $\left( -4;\, 4\right) $

Bài 2. Cho phương trình: $ 2x^2-5x-3=0 $ có 2 nghiệm $ x_1; x_2 $.

Không giải phương trình, hãy tính giá trị của biểu thức: $ A=(x_1+2x_2)(x_2+2x_1) $.

Giải

Ta có: $\Delta = \left( -5\right) ^2-4\cdot 2\cdot (-3)=49>0\Rightarrow $ Phương trình có hai nghiệm phân biệt.

Áp dụng định lý Viete ta có: $\left\{ \begin{array}{l} S=x_1+x_2=\dfrac{5}{2}\\ P=x_1x_2=-\dfrac{3}{2} \end{array} \right. $

Ta có: $A=\left( x_1+2x_2\right) \left( x_2+2x_1\right)=2\left( x_1^2 +x_2^2\right) +5x_1x_2=2\left( S^2-2P\right) +5P=11 $

Vậy $A=11$

Bài 3. Quy tắc sau đây cho ta biết CAN, CHI của năm X nào đó.

Để xác định CAN, ta tìm số dư $ r $ trong phép chia $X$ cho $10$ và tra vào bảng $1$.

Để xác định CHI, ta tìm số dư $ s $ trong phép chia $X$ cho $12$ và tra vào bảng $2$.

Ví dụ: năm $2020$ có CAN là Canh, có CHI là Tí.

Bảng 1

Bảng 2

a) Em hãy sử dụng quy tắc trên để xác định CAN, CHI của năm $2005$?

b) Bạn Hằng nhớ rằng Nguyễn Huệ lên ngôi hoàng đế, hiệu là Quang Trung vào năm Mậu Thân nhưng không nhớ rõ đó là năm bao nhiêu mà chỉ nhớ là sự kiện trên xảy ra vào cuối thế kỉ $18$. Em hãy giúp Hằng xác định chính xác năm đó là năm bao nhiêu?

Giải

a) Năm $2005$ có CAN là Ất, có CHI là Dậu.

b) Vì năm hoàng đế Nguyễn Huệ lên ngôi là cuối thế kỉ $18$ nên năm đó có dạng $\overline{17ab}$ với $a,\ b\in \mathbb{N}$ và $0\le a,\ b\le 9$

Năm đó có CAN là Mậu nên ta có $\overline{17ab}$ chia $10$ dư $8$ suy ra chữ số tận cùng $b=8$

Năm đó có CHI là Thân nên ta có $\overline{17a8}$ chia hết cho $12$. Suy ra $\overline{17a8}$ chia hết cho $3$.

Khi đó: $1+7+a+8= 16+a\ \vdots \ 3 \Rightarrow a\in \left\{ 2;\ 5;\ 8\right\} $

Với $a=2\Rightarrow 1728$ chia $10$ dư $8$ và $1728$ chia $12$ dư $0$.

Với $a=5\Rightarrow 1758$ chia $10$ dư $8$ và $1758$ chia $12$ dư $6$ (loại).

Với $a=8\Rightarrow 1788$ chia $10$ dư $8$ và $1788$ chia $12$ dư $0$.

Vì sự kiện xảy ra vào cuối thế kỉ $18$ nên năm đó là năm $1788$.

Bài 4. Cước điện thoại $ y $ (nghìn đồng) là số tiền mà người sử dụng điện thoại cần trả hàng tháng, nó phụ thuộc vào lượng thời gian gọi $ x $ (phút) của người đó trong tháng. Mối liên hệ giữa hai đại lượng này là một hàm số bậc nhất $ y=ax+b. $ Hãy tìm $ a,b $ biết rằng nhà bạn Nam trong tháng $5$ đã gọi $100$ phút với số tiền là $40$ nghìn đồng và trong tháng $6$ đã gọi $40$ phút với số tiền $28$ nghìn đồng.

Giải

Với $x=100$ và $y=40$ ta có $40=100a+b$

Với $x=40$ và $y=28$ ta có $28=40a+b$

Ta có hệ phương trình: $\left\{ \begin{array}{l} 100a+b=40\\ 40a+b=28 \end{array}\right. $ $\Leftrightarrow \left\{ \begin{array}{l} a=\dfrac{1}{5}\\ b=20  \end{array}\right. $

Vậy $a=\dfrac{1}{5}$ và $b=20$

Bài 5. Theo quy định của cửa hàng xe máy, để hoàn thành chỉ tiêu trong $1$ tháng, mỗi nhân viên phải bán được trung bình một chiếc xe máy trong một ngày. Nhân viên nào hoàn thành chỉ tiêu trong một tháng thì nhận được lương cơ bản là $8 000 000$ đồng. Nếu trong tháng nhân viên nào bán vượt chỉ tiêu thì được hưởng thêm $8\%$ tiền lời của số xe máy bán vượt chỉ tiêu đó. Trong tháng $5$ (có $31$ ngày), anh Thành nhận được số tiền là $9 800 000$ đồng (bao gồm cả lương cơ bản và tiền thưởng thêm của tháng đó). Hỏi anh Thành đã bán được bao nhiêu chiếc xe máy trong tháng $5$, biết rằng mỗi xe máy bán ra thì cửa hàng thu lời được $2 500 000$ đồng.

Giải

Tháng $5$ có $31$ ngày nên số xe máy tiêu chuẩn phải bán được là $31$ xe.

Gọi $x$ ($x>0$) là số xe máy anh Thành đã bán vượt chỉ tiêu.

Số tiền anh Thành được thưởng thêm là: $8\% \cdot 2\, 500\, 000 \cdot x = 200\, 000x$

Ta có phương trình: $200\, 000x = 9\, 800\, 000 -8\, 000\, 000 \Rightarrow x=9$

Vậy anh Thành đã bán được $40$ xe máy trong tháng $5$.

Bài 6. Anh Minh vừa mới xây một cái hồ trữ nước cạnh nhà có hình dạng hộp chữ nhật có kích thước $2m \times 2 m \times 1 m$. Hiện hồ chưa có nước nên anh Minh phải ra sông lấy nước. Mỗi lần ra sông anh gánh được $1$ đôi nước đầy gồm $2$ thùng hình trụ bằng nhau có bán kính đáy $0,2 \ m$, chiều cao $0,4 \ m$.

a)Tính lượng nước ($m^3$) anh Minh đổ vào hồ sau mỗi lần gánh (ghi kết quả làm tròn đến $2$ chữ số thập phân). Biết trong quá trình gánh nước về thì lượng nước bị hao hụt khoảng $10\%$ và công thức tính thể tích hình trụ là $V = \pi R^2h$.

b) Hỏi anh Minh phải gánh ít nhất bao nhiêu lần để đầy hồ? Bỏ qua thể tích thành hồ.

Giải

a) Thể tích nước anh gánh được trong hai thùng là: $V = 2\cdot \pi R^2h =\dfrac{4\pi }{125}$ ($m^3$)

Lượng nước anh Minh đổ vào hồ sau mỗi lần gánh là: $90\% \cdot V=\dfrac{18\pi }{625}\approx 0,09$ ($m^3$)

b) Thể tích hồ trữ nước là: $V_{\text{hồ}}=2\cdot 2\cdot 1=4$ ($m^3$)

Ta có: $\dfrac{V_{\text{hồ}}}{V}\approx 44,21 $

Vậy anh Minh phải gánh ít nhất $45$ lần để đổ nước đầy hồ.

Bài 7. Sau buổi sinh hoạt ngoại khóa nhóm bạn của Thư rủ nhau đi ăn kem ở một quán gần trường. Do quán mới khai trương nên có khuyến mãi, bắt đầu từ ly thứ $5$ giá mỗi ly kem được giảm $1 500$ đồng so với giá ban đầu. Nhóm của Thư mua $9$ ly kem với số tiền là $154 500$ đồng. Hỏi giá của một ly kem ban đầu?

Giải

Gọi $x$ (đồng) là giá tiền của một ly kem khi chưa giảm (ĐK: $x \geq 1 500$ đồng)

$ \Rightarrow$ Giá tiền ly kem từ ly thứ 5 trở đi là: $x-1 500$ (đồng)

Theo bài ra ta có:  $4.x+5 (x- 1500)= 154 500 \Leftrightarrow x=18 000$ ( đồng)

Vậy giá tiền ly kem ban đầu là: $18000$ đồng.

Bài 8. Cho đường tròn tâm $O$; bán kính $R$ và điểm $A$ nằm ngoài đường tròn sao cho $OA>2R$. Từ $A$ kẻ $2$ tiếp tuyến $AD$; $AE$ đến đường tròn ($O$) ($D$; $E$ là hai tiếp điểm). Lấy điểm $M$ nằm trên cung nhỏ $DE$ sao cho $MD >ME$. Tiếp tuyến của đường tròn (O) tại $M$ cắt $AD$; $AE$ lần lượt tại $I$; $J$. Đường thẳng $DE$ cắt $OJ$ tại $F$ .

a) Chứng minh: $OJ$ là đường trung trực của đoạn thẳng $ME$ và $\angle OMF=\angle OEF$.

b) Chứng minh: tứ giác $ODIM$ nội tiếp và $5$ điểm $I;\ D; \ O;\ F;\ M$ cùng nằm trên một đường tròn.

c) Chứng minh: $\angle JOM=\angle IOA$ và $\sin \angle IOA=\dfrac{MF}{IO}$

Giải

a)

  • Ta có: $ \left\lbrace \begin{array}{l} OM=OE (=R)\\ MJ=EJ \end{array} \right. \Rightarrow OJ$ là đường trung trực của đoạn $ME$
  • Ta có: $OJ$ là tia phân giác của góc $\angle EOM \Rightarrow \angle EOJ =\angle MOJ$

Xét $\triangle EOF $ và $\triangle MOF$ ta có: $OF$ chung, $OM=OE$, $\angle EOF= \angle MOF$

$\Rightarrow \triangle EOF = \triangle MOF \Rightarrow \angle OMF =\angle OEF$.

b)

  • Tứ giác $ODIM$ có: $\angle ODI +\angle OMI =90^{\circ} +90^{\circ}= 180^{\circ}$

$\Rightarrow $ Tứ giác $ODIM$ là tứ giác nội tiếp $(1)$.

  • Ta có: $\angle ODE =\angle OED$ và $\angle OEF =\angle OMF$

$ \Rightarrow \angle ODF =\angle OMF \Rightarrow $ Tứ giác $ODMF$ là tứ giác nội tiếp $(2)$.

Từ $(1)$, $(2)$ ta có: $5$ điểm $I,D,O,F,M$ cùng thuộc một đường tròn.

c)

  • Tứ giác $IDFM$ nội tiếp nên ta có: $\angle IOF =\angle IDF = \dfrac{1}{2}$ sđ cung $IF$ $(3)$

Tứ giác $ADOE$ nội tiếp nên : $\angle ADE =\angle AOE$ $(4)$

Từ $$(3)$, $(4)$ ta có: $\angle IOF =\angle AOE$

Mà ta có: $ \angle IOF =\angle IOA+ \angle AOF$

$ \angle AOE=\angle AOF +\angle EOF$

Suy ra: $ \angle EOF =\angle IOA$

Mặt khác $\angle EOF =\angle JOM$ ( do $OJ$ là tia phân giác$ EOM$ )

Vậy $\angle JOM =\angle IOA$ $(5)$

  • Ta có: $\triangle JMF \backsim \triangle JOI$ (g.g) $\Rightarrow \dfrac{JM}{JO} =\dfrac{MF}{OI}$ $(6)$

Xét tam giác $OMJ$ vuông tại $M$ nên: $\sin \angle JOM =\dfrac{MJ}{OJ}$ (7)

Từ $(5), (6), (7)$ suy ra: $ \sin \angle IOA=\dfrac{MF}{IO}$